0% found this document useful (0 votes)
51 views117 pages

First Year Math Batch

Uploaded by

Luke Mpasela
Copyright
© © All Rights Reserved
We take content rights seriously. If you suspect this is your content, claim it here.
Available Formats
Download as PDF, TXT or read online on Scribd
0% found this document useful (0 votes)
51 views117 pages

First Year Math Batch

Uploaded by

Luke Mpasela
Copyright
© © All Rights Reserved
We take content rights seriously. If you suspect this is your content, claim it here.
Available Formats
Download as PDF, TXT or read online on Scribd
You are on page 1/ 117

SETS

Important things to know

 Definition of a set
 Set operations
 De Morgan’s laws
 Rational numbers
 Irrational numbers
 Complex numbers
 Surds
Definition: A set is a collection of distinct and well defined objects. There are two ways of representing
sets, description and listing. These are self-explanatory.

Definition: Two sets A and B are said to be equal if and only if every element of A is an element
of B and every element of B is an element of A. That is, they have same elements. That is, if
A  B and B  A  A  B

Definition: A set A is a subset of a set B, written, A  B , if every element of A is also an element


of B.
Stop and think: are all empty sets equal? If you are confused, get back to the definition and try
to read it again.
Operations on sets.

Notations
 = union,  = intersection, A' = A compliment, which means all elements in the universal set
except those in A,  = empty set or   = empty set. We use one of these two symbols and not
both.
Just as numbers are added, subtracted, sets can also be operated under union, intersection and
complements of sets between two or more sets.
Intersection of sets: Notation: A  B , reads A intersection B
Definition: Intersection: is a set of elements which are found in both set A and set B.

A  B is a set of elements found in both A and B

1
The intersection of sets A and B contains a particular group of elements that are found in both
set A and in set B.

Union of sets: Notation; A  B read, A union B


Definition: Union is a set of elements which are found in A or B or in both A and B ) note that
when writing the elements of the union of two or more sets, each element is written once even if
it appears in both or in more sets.

A  B is a set of elements which are found in A or in B or in both A and B ( note that elements
found in both A and B are written once).

Complement: Notation; A ' reads, A complement

Definition: A ' is the set of elements in the universal set except elements in A.

A' (A-dash) is called the complement of A. It contains all elements which are in the universal set
but not members of A. A and A’ added together make up the Universal set. That is A  A'  U

Example 1.3.3

Let u  a, e, i, o, u and subsets A  a, e, u and B  a, i, o

Find
(i) A B (ii) A B (iii) A' B'

(iv) ( A  B )'

2
Solutions
(i) List the elements found in both A and B
A  B  a , note that a is the only element found in both A and B

(ii) Here we list all the elements found in A and those elements found in B
A  B  a, e, i, o, u , note that ‘a’ which appears in both A and B it is written once.

(iii) List the elements of A’ and B’ and write the elements found in both A’ and B’

A'  i, o and B '  e, u then A' B '    means there is no element common in A’ and B’.

(iv) List the elements found in A and B then find the complement of this set.
A  a, e, uand B  a, i, othen, A  B  a, e, i, o, u , ( A  B )'  

3
Laws of the algebraic of sets

1. a) A  A  A b. A A  A Idempotent laws

2 a) ( A  B )  C  A  ( B  C ) b. ( A  B )  C  A  ( B  C ) Associative laws

3 a) A  B  B  A b. A B  B  A Commutative laws

4 a) A  ( B  C )  ( A  B )  ( A  C )

b) A  ( B  C )  ( A  B )  ( A  C ) Distributive laws

5 a) A    A b) A U  A Identity laws

6. a) A  U  U b) A  A Identity laws

7 a) A  A '  U b. A  A'   Compliment laws

8 a) ( A ' ) '  A b. U '  , ' U Compliment laws

9 a) ( A  B)  A '  B ' b. ( A  B)  A '  B ' De Morgan’s laws

NOTE: These laws are very important and useful, make sure you know them. Always
examinable, especially on set distributions and simplifications. Also notice how compliment
behaves when it meets other operations in brackets.

De Morgan’s law:
The complement of the union of two sets is equal to the intersection of their
complements and the complement of the intersection of two sets is equal to the
union of their complements. These are called De Morgan’s laws.

For any two finite sets A and B;

(i) (A U B)' = A' ∩ B' (which is a De Morgan's law of union).

4
(ii) (A ∩ B)' = A' U B' (which is a De Morgan's law of intersection).

Proof of De Morgan’s law: (A U B)' = A' ∩ B'

Let P = (A U B)' and Q = A' ∩ B'

Let x be an arbitrary element of P then x ∈ P ⇒ x ∈ (A U B)'

⇒ x ∉ (A U B)

⇒ x ∉ A and x ∉ B

⇒ x ∈ A' and x ∈ B'

⇒ x ∈ A' ∩ B'

⇒x∈Q

Therefore, P ⊂ Q …………….. (i)

Again, let y be an arbitrary element of Q then y ∈ Q ⇒ y ∈ A' ∩ B'

⇒ y ∈ A' and y ∈ B'

⇒ y ∉ A and y ∉ B

⇒ y ∉ (A U B)

⇒ y ∈ (A U B)'

⇒y∈P

Therefore, Q ⊂ P …………….. (ii)

Now combine (i) and (ii) we get; P = Q i.e. (A U B)' = A' ∩ B'

Proof of De Morgan’s law: (A ∩ B)' = A' U B'

Let M = (A ∩ B)' and N = A' U B'

Let x be an arbitrary element of M then x ∈ M ⇒ x ∈ (A ∩ B)'

⇒ x ∉ (A ∩ B)

⇒ x ∉ A or x ∉ B

5
⇒ x ∈ A' or x ∈ B'

⇒ x ∈ A' U B'

⇒x∈N

Therefore, M ⊂ N …………….. (i)

Again, let y be an arbitrary element of N then y ∈ N ⇒ y ∈ A' U B'

⇒ y ∈ A' or y ∈ B'

⇒ y ∉ A or y ∉ B

⇒ y ∉ (A ∩ B)

⇒ y ∈ (A ∩ B)'

⇒y∈M

Therefore, N ⊂ M …………….. (ii)

Now combine (i) and (ii) we get; M = N i.e. (A ∩ B)' = A' U B'

Examples on De Morgan’s law:

1. If U = {j, k, l, m, n}, X = {j, k, m} and Y = {k, m, n}.

Proof of De Morgan's law: (X ∩ Y)' = X' U Y'.

Solution:

We know, U = {j, k, l, m, n}

X = {j, k, m}

Y = {k, m, n}

(X ∩ Y) = {j, k, m} ∩ {k, m, n}

= {k, m}

Therefore, (X ∩ Y)' = {j, l, n} ……………….. (i)

Again, X = {j, k, m} so, X' = {l, n}

and Y = {k, m, n} so, Y' = {j, l}

X' ∪ Y' = {l, n} ∪ {j, l}

6
Therefore, X' ∪ Y' = {j, l, n} ……………….. (ii)

Combining (i)and (ii) we get;

(X ∩ Y)' = X' U Y'. Proved

2. Let U = {1, 2, 3, 4, 5, 6, 7, 8}, P = {4, 5, 6} and Q = {5, 6, 8}.


Show that (P ∪ Q)' = P' ∩ Q'.

Solution:

We know, U = {1, 2, 3, 4, 5, 6, 7, 8}

P = {4, 5, 6}

Q = {5, 6, 8}

P ∪ Q = {4, 5, 6} ∪ {5, 6, 8}

= {4, 5, 6, 8}

Therefore, (P ∪ Q)' = {1, 2, 3, 7} ……………….. (i)

Now P = {4, 5, 6} so, P' = {1, 2, 3, 7, 8}

and Q = {5, 6, 8} so, Q' = {1, 2, 3, 4, 7}

P' ∩ Q' = {1, 2, 3, 7, 8} ∩ {1, 2, 3, 4, 7}

Therefore, P' ∩ Q' = {1, 2, 3, 7} ……………….. (ii)

Combining (i)and (ii) we get;

(P ∪ Q)' = P' ∩ Q'. Proved

EXAMPLES
( A  B )'  A' B ' and ( A  B )'  A' B '

We can verify de Morgan’s laws by showing that the LHS = RHS of the De Morgan’s law.

Example 1 Let U  0,1,2,3,4,5,6,7,8,9,10, and A  set of even numbers less than 10 , and B= set
of prime numbers less than 9. Verify the De Morgan’s laws:

7
(i) ( A  B )'  A' B '
(ii) ( A  B )'  A' B '
Solutions

 You find list the elements of A and B first as; A  0,2,4,6,8 , B  2,3,5,7 and we then
find A  B  2 and we find the complement of this intersection
( A  B )'  0,1,3,4,5,6,7,8,9,10 .
Then you find A complement and B complement as: A' 1,3,5,7,9,10 and
B ' 0,1, ,4,6,8,9,10 , then we find A'B ' )  0,1,3,4,5,6,7,8,9,10
Now you can see that ( A  B )'  A' B ' = 0,1,3,4,5,6,7,8,9,10
Hence, De Morgan’s law has been verified.
 You should try ( A  B )'  A' B '

TIP: De Morgan’s laws are very important, always examinable. You must know them
by heart, both stating, proving and applications.

Disjoint Sets: Two sets A and B are said to be disjoint sets if and only if their intersection is
empty. That is, A  B  

Example Let U = the set of positive counting numbers, be the Universal set, and A = the set of
even numbers and B = be the set of odd numbers. Find A  B ?

Solution We know that there is no number which is both even and odd, so A  B  

Equivalent sets: Two sets A and B are said to be equivalent sets if and only if they have the
same number of elements, and these elements are not the same.
Example 1.3.6 Let A = { a, b , c , d} and B= { 1,2,3,4}
The two sets are equivalent and we write A  B

Sets of Numbers
Notations and symbols

N  1,2,3,4,....., the set of natural numbers or counting numbers

W  0,1,2,3,4,5,....... , the set of whole numbers

Z  ....,3,2,1,0,1,2,3,4,...... , the set of integers. Note that the set of integers has three sets of
numbers, namely the negative numbers, zero and positive numbers.
What this means is the this set contains the set of whole numbers W, and the set of Natural
numbers N. in symbols we write, N  W  Z . The set of positive integers can be written as:

8
Z   1,2,3,4,5,.....and the set of negative integers can be written as: Z   ........  3,2,1 , the
two set plus 0 gives the set of integers.
The next set to this set of integers is called the set of rational numbers.

a 
Definition 1: A rational number is defined as ; Q   , b  0, where a , b  R  , that is the set of
b 
a
rational numbers can be written in the form , b  0 where a and b are integers. Note that in
b
mathematics division by 0 is not allowed, therefore, where an unknown appears in the
denominator, it is only valid in a case where it assume that the value is not zero in the
a
denominator. A number in the form can be expressed in decimal form, and two possibilities
b
1 3 2
arises: (1) either the decimal terminates such as:
, , , etc. These have terminal decimals.
2 4 5
1 22
(2) Repeated or non-terminal decimals such as:  0.3333 ........ ,  3.1428571428 57 .....
3 7

This set of rational numbers contains all integers, in symbols we write: Z  Q , since whole
a 3
numbers can be written in the form , for example, 3  , hence it is a rational number by
b 1
definition.
Definition 2: A rational number can be written as a terminating decimal or decimals repeat

3 1
For example  0 .75 , this is an example of terminating decimals,  0.33333 ....... , this is an
4 3
example of repeating decimals. This type of decimals can be expressed in another shorter form,

1
that is,  0.3333 ......  0. 3 , the bar on top shows that 3 repeats.
3

3
What is known is that o.75 can easily be converted to a fraction which is . What would be
4

interesting to here is how 0.3333………. can be converted to a fraction. Let a  0. 3 , multiply 10
both side of the equation. We multiply by 10 because the repeating decimal has on digit after the
decimal point. If the repeating digits were two we would have multiplied by 100.

10a  3. 3 ……(1)

a  0. 3 …….(2)

Subtract (2) from (1)  9 a  3 , Divide by 9 both sides

9
3 1
a  , a 
9 3
___ _
Other examples are: 2.412 means 2.412121212..... , that is 12 repeats, 1.543 7 =1.54377777....

a
If a number cannot be written in the form , b  0 where a and b are integers or cannot be
b
written as terminating decimals or repeated decimals, then this number is an Irrational
number(I). That is, Q  I   , but Q  I  R ,that is the set of rational numbers plus the set of
irrational numbers equal the set of Real numbers. That is the set of real numbers contains rational
numbers and irrational numbers. That is, a number is either a rational number or an irrational
number and not both. In the following example we use this principle which states that if a
number is not rational, then it must be irrational.

Example 1.4.1 Prove that 2 is irrational.

a
Proof :We assume that 2 is rational. Since it is rational, then we can write it in the form ,
b
b  0 . Note that this assumption is what will help us to conclude as to whether the number is
a a
rational or irrational. That is 2 , b  0 , Take the equation 2  , equation (1),
b b
where a and b are prime ( that mean a and b have no common factor). And square both sides,
and we get

a2
2  2 , 2b 2  a 2 , (2)
b

This implies a 2 is divisible by 2, then a is divisible by 2. That is a has a factor 2 say a  2r , for
some integer r. We substitute a = 2r in (2), and we get;

2b 2  (2r ) 2 , this implies, 2b 2  4 r 2 , b 2  2r 2 , this implies b 2 is divisible by 2, then b is


divisible by 2. Now we have found a common factor 2 this is a contradiction by (1)

Therefore, 2 is not rational, hence 2 is irrational.

Note that the set of real numbers is the union of the set of rational number and the set of
irrational number. This implies that R  Q  I , a real number is either rational or irrational, that
is Q  I   . The set of real numbers are closed on the number line.

Now that you have followed well the whole procedure, try to do the next example before
checking at what we have done for you. This will help you to consolidate the understanding of
the concept.

Example 1.4.2 Prove that 3 is not rational?

10
a
Proof :Assume that 3 is rational. If it is rational, then it can be expressed in the form were
b
a, b are integers, and prime ( meaning they have no common factors). But any integer can be
 3n  9n 2
 
written in the form 3n   3n  1 for n  Z  9 n 2   9 n 2  6 n  1 equation (1)
3n  2 9 n 2  12 n  4
 

Therefore, we can now write 3 as a rational number by the assumption.

a
3 , where b  0 , and a and b are integers. We square both
b
sides, and obtain,

a2
3 2
b equation (2)
3b  a 2
2

Therefore, a 2 is a divisible by 3, then by (1) a is divisible by 3, this implies a has a factor 3. We


equate a  3n and substitute it in equation (2). Then

3b 2  (3n ) 2
3b 2  9 n 2
b 2  3n 2

This implies, b 2 is a divisible by 3, then by (1) above b is divisible by 3, this implies b has a
factor 3.
This is a contradiction since our assumption is that a and b are prime, but we have found a
common factor 3. Therefore, our assumption is false, that is 3 , is not rational.

Hence, 3 is irrational. NOTE: Irrationality is always examinable, atleast three times in a


year, quiz 1, test 1 and in the final exam. It carries a lot of marks, make sure you master it and
get it right at all times.
Example 1.4.2 Find an irrational number between the interval (a, b).
Solution. Note that a and b can be two real numbers such that b > a. If this condition is not
satisfied, the method demonstrated here may not work. Begin by finding the interval length of
this interval. And assume that we know that 2 is an irrational number. (which can be proved as
in example 1.4.1).

b  a  0 , we divide this 2 and we have.

11
ba
0  b  a here we form an inequality, then we add a throughout and the
2
inequality is maintained.

ba
0a   a b  a  a
2
ba
a ab
2

ba
We know that is an irrational number
2

ba
Therefore the required irrational number is a
2

ADDITIONAL QUESTIONS AND SOLUTIONS


Question 1.

a) Prove that √10 is irrational.

Solution: assume 10 is rational for a contradiction. Then there are a ∈ Z, b ∈ N such that
and a,b have no factors in common.

Squaring both sides and rearranging gives 10b2 = a2. Therefore both 5 and 2 are factors of a2, and
hence of a (using the argument given in lectures). Using the Fundamental Theorem of
Arithmetic, there is an integer n such that a = 10n. Putting this into 10b2 = a2 gives (on cancelling
factors of 10) b2 = 10n2. Therefore b has a factor of 10 (because b2 does), so a and b have a factor
in common. This is a contradiction, and so√ 10 is irrational.

b) Hence prove that 2+ 5 is irrational. (Hint: Assume by contradiction that ,


square the equation and then try to simplify first.) Solution: following the hint, we get

,
from which it follows that

The right-hand side of this equation is a rational number, implying that


√10 ∈√Q. But this contradicts part (a), from which we conclude that √2 + √5 is irrational.

12
Real numbers
You can represent a subsets of real numbers in interval form such as: (a,b) is an open interval,
[a,b] as a closed interval, (a,b] as an open-closed interval, and [a,b) as a closed- open interval. In
lower mathematics these sets where represented using set builder notation for instance
( a, b)  x : a  x  b, x  R , while [ a, b]  x : a  x  b, x  R, and these can be represented on a
number line as illustrated below.

Example 1.4.2 Given that the universal set is R, A  (1,4] and B  [ 0,7 ] . Find the following
sets and illustrate them on the number line. (i) A' (ii) A  B (iii) A'B

Solutions
(i) You draw a number line to help you visualize the given set. Note that the interval
(-1, 4] is open-closed interval and this can be illustrated on a number line as follows:

Therefore A'  (  ,1]  ( 4,  )


(ii) The shaded part in the number line is the intersection of A and B as shown below:

That is A  B  [0,4]
1.4.1 Some properties of real numbers
1. Identity element.
0 is an identity element under the operation +; 0+4=4
1 is an identity element under the operation x, 1x4=4

2. commutative properties
The operation + is commutative; a+b=b+a, for real numbers a and b
The operation x is commutative; axb = bxa , for real numbers a and b
3. Inverse element
The operation + has an inverse element 0, for real number a
The operation x has an inverse element a 1 , for real number a
4. Transitive property
if a  b and b  c ,then a  c
5. Associative property
The operation + is associative

13
The operation x is associative
6. The distributive property
The operation x is distributive over +: a(b+c)=ab+ac

Radicals
We’ll start this section by defining a radical. If n is a positive integer that is greater than 1 and a
1

is a real number then, n


a  a n where n is called the index, a is called the radicand, and the
symbol is called the radical. The left side of this equation is called the radical form and the
right side is called the exponent form. The left side of this equation is often called the radical
form and the right side is often called the exponent form. From this definition we can see that a
radical is simply another notation for the first rational exponent that we looked at in the rational
exponents section. Note as well that the index is required in these to make sure that we correctly
evaluate the radical. There is one exception to this rule and that is square root. For square roots
we have, 2
a  a that is for square root the 2 is dropped and you just write square root of a.

Example 1 Write each of the following radicals in exponent form.

4
(a) 16
10
(b) 8x
(c) x2  y2

14
Solutions
1

(a) 4
16  (16 ) 4
1

(b) 10
8 x  (8 x ) 10

(c) x2  y 2  (x2  y 2 ) 2
As seen in the last two parts of this example we need to be careful with parenthesis. When we
convert to exponent form and the radicand consists of more than one term then we need to
enclose the whole radicand in parenthesis as we did with these two parts. To see why this is
1

consider the following, 8x 10 From our discussion of exponents in the previous sections we know
that only the term immediately to the left of the exponent actually gets the exponent. Therefore,
1

the radical form of this is, 8 x 10


 810 x  10 8 x

So, we once again see that parenthesis are very important in this class. Since we know how to
evaluate rational exponents we also know how to evaluate radicals.

Example 2 Evaluate each of the following.

(a) 16 and 4 16
5
(b) 243
(c) 3
 125
(d) 4 16
Solution
1 1 1 1

(a) 16  (16 ) 2  ( 4 2 ) 2  4 and 4


16  (16 ) 4  ( 2 4 ) 4  2
1 1

(b) 5
243  ( 243 ) 5  (35 ) 5  3
1 1

(c) 3
 125  ( 125 )  (( 5) )  5
3 3 3

(d) 4
 16  ( 16 ) 4 This cannot be evaluated since there is no real number which be found
under the fourth root of -16.
Note however that we can evaluate the radical of a negative number if the index is odd as the
previous part shows.

We can also write the general rational exponent in terms of radicals as follows.
m 1 m 1

a n  (a m ) n  n a m or a n  (a n ) m  (n a ) m

15
m 1 m 1

a n  (a n ) m  (n a ) m or a n  (a m ) n  n a m

Properties

If n is a positive integer greater than 1 and both a and b are positive real numbers then,

(a) n
an  a
(b) n
ab  n a n b
n
a a
(c) n 
n
b
b
Note that on occasion we can allow a or b to be negative and still have these properties
work. When we run across those situations we will acknowledge them. However, for the
remainder of this section we will assume that a and b must be positive.

Also note that while we can “break up” products and quotients under a radical we can’t do the
same thing for sums or differences. In other words,

For example ab  a  b

If we “break up” the root into the sum of the two pieces we clearly get different answers! So, be
careful to not make this very common mistake!

We are going to be simplifying radicals shortly so we should next define simplified radical
form. A radical is said to be in simplified radical form (or just simplified form) if each of the
following are true.

1. All exponents in the radicand must be less than the index.


2. Any exponents in the radicand can have no factors in common with the index.
3. No fractions appear under a radical.
4. No radicals appear in the denominator of a fraction.

Example 3 Simplify each of the following. Assume that x, y, and z are positive.

(a)

(b)

(c)

16
(d)

(e)

(f)

Solution

(a) In this case the exponent (7) is larger than the index (2) and so the first rule for
simplification is violated. To fix this we will use the first and second properties of
radicals above. So, let’s note that we can write the radicand as follows.
y7  y6 y  ( y3 )2 y
So, we’ve got the radicand written as a perfect square times a term whose exponent is smaller
than the index. The radical then becomes, y 7  ( y 3 ) 2 y . Now use the second property of
radicals to break up the radical and then use the first property of radicals on the first term.
y7  ( y3 )2 y  y3 y

This now satisfies the rules for simplification and so we are done.

How do we figure out how to break up the exponent as we did? To do this we noted that the
index was 2. We then determined the largest multiple of 2 that is less than 7, the exponent on the
radicand. This is 6. Next, we noticed that 7=6+1.

(b) This radical violates the second simplification rule since both the index and the exponent
have a common factor of 3. To fix this all we need to do is convert the radical to exponent form
do some simplification and then convert back to radical form.

1 6 2 1
9
x  (x )  x  x  (x )  3 x2
6 6 9 9 3 2 3

(c) There is more than one term here but everything works in exactly the same fashion. We
will break the radicand up into perfect squares times terms whose exponents are less than 2 (i.e.
1).

18 x 6 y 11  9 x 6 y 10 (2 y )  9( x 3 ) 2 ( y 5 ) 2 (2 y ) . Don’t forget to look for perfect squares in the number


as well. Now, go back to the radical and then use the second and first property of radicals as we
did in the first example.

17
18 x 6 y 11  9( x 3 ) 2 ( y 5 ) 2 ( 2 y )  9 ( x 3 ) 2 2 y  3 x 3 y 5 2 y

Note that we used the fact that the second property can be expanded out to as many terms as we
have in the product under the radical. Also, don’t get excited that there are no x’s under the
radical in the final answer. This will happen occasionally.

(d) This one is similar to the previous part except the index is now a 4. So, instead of get
perfect squares we want powers of 4. This time we will combine the work in the previous part
into one step.

4
32 x 9 y 5 z 12  16 x 8 y 4 z 12 ( 2 xy )  4 16 4 ( x 2 ) 4 4 y 4 4 ( z 3 ) 4 4 2 xy  2 x 2 yz 3 4 2 xy

(e) Again this one is similar to the previous two parts.

5
x 12 y 4 z 24  5 x 10 z 20 ( x 2 y 4 z 4 )  5 ( x 2 ) 5 5
(z 4 )5 5
x2 y4z4  x2z4 5 x2 y4z4

(f) Individually both of the radicals are in simplified form. However, there is often an
unspoken rule for simplification. The unspoken rule is that we should have as few radicals in the
problem as possible. In this case that means that we can use the second property of radicals to
combine the two radicals into one radical and then we’ll see if there is any simplification that
needs to be done.

3
9x 2 3
6 x 2  3 (9 x 2 )( 6 x 2 )  3 54 x 4

Now that it’s in this form we can do some simplification.

3
9x 2 3
6 x 2  3 27 x 3 ( 2 x )  3 27 x 3 ( 2 x )  3 x 3
2x

If we are looking at the product of two radicals with the same index then all we need to do is use
the second property of radicals to combine them then simplify. What we need to look at now are
problems like the following set of examples.

Before you get excited that you have understood radicals, try to do the following example
without referring to the solution given, if you it correct, the be proud of yourself, be happy,
you are a genius.

Example 4 Multiply each of the following. Assume that x is positive.

(a) ( x  2)( x  5)

(b) (3 x  
y) 2 x  5 y 

18
(c) (5 x  2)( 5 x  2)

Solutions

(a) ( x  2)( x  5)  x  5 x  2 x  10 y
= x  3 x  10
 
(b) (3 x  y ) 2 x  5 y  6 x  15 xy  2 xy  5 y
= 6 x  17 xy  5 y
(c) (5 x  2)( 5 x  2)  25 x  10 x  10 x  4
= 25 x  4
We look at some simplification examples illustrating the final two rules. Note as well that the
fourth rule says that we shouldn’t have any radicals in the denominator. To get rid of them we
will use some of the multiplication ideas that we looked at above and the process of getting rid of
the radicals in the denominator is called rationalizing the denominator. In fact, that is really
what this next set of examples is about. They are really more examples of rationalizing the
denominator rather than simplification examples.

Example 5 Rationalize the denominator for each of the following. Assume that x is positive.

1
(a)
x

(b) 3
x2

3
(c)
1 x

1
(d)
x 3

2
(e)
x 2

Solutions

(a)

Rationalizing the denominator may seem to have no real uses in an Algebra class
now. However, as you progress in calculus class you will find that rationalizing is useful on
occasion at that level. Recall that when we first wrote down the properties of radicals we

19
required that ‘a’ be a positive number. This was done to make the work in this section a little
easier. However, with the first property that doesn’t necessarily need to be the case.

Here is the property for a general a (i.e. positive or negative)

 a if n is even
n
an  
 a if n is odd

Where a is the absolute value of a. All that you need to do is know at this point is that absolute
value always makes a a positive number.

So, as a quick example this means that,

6
x6  x and 7
x7  x

For square roots this is,

x2  x

Surds
Surds are numbers left in root form (√) to express its exact value. It has an infinite number of
non-recurring decimals. Therefore, surds are irrational numbers.
There are certain rules that we follow to simplify an expression involving surds. Rationalising
the denominator is one way to simplify these expressions. It is done by eliminating the surd in
the denominator. This is shown in Rules 3, 5 and 6.
It can often be necessary to find the largest perfect square factor in order to simplify surds. The
largest perfect square factor is found by looking at any possible factors of the number that is
being square rooted. Lets say that you are looking at the square root of 242. Can you simplify
this? Well, 2 x 121 is 242 and we can take the square root of 121 without leaving a surd (because
we get 11). Since we cannot take the square root of a larger number that can be multiplied by
another to give 242 then we say that 121 is the largest perfect square factor.

Six Rules of Surds


Rule 1:
An Example:
Simplify :
Since , as 9 is the largest perfect square factor of 18.

20
Rule 2:

An Example:

Simplify :

Rule 3:
By multiplying both the numberator and denominator by the denominator you can rationalise the
denominator.
An Example:
rationalise :

Rule 4:
An Example:
Simplify :

Rule 5:
Following this rule enables you to rationalise the denominator.
An Example:

21
Rationalise :

Rule 6:
Following this rule enables you to rationalise the denominator.
An Example:
Rationalise :

You have now learnt the important rules of surds.


TIP: These 6 rules of surds if well understood, you can never fail any
question on surds. There has never been an exam without surds. This is
the easiest way one can help you understand surds for they are very
important in mathematics.

Complex Numbers
In the radicals section we noted that we won’t get a real number out of a square root of a

negative number. For instance  4 isn’t a real number since there is no real number that we
can square and get a negative 4. But we have some imaginations of what the answer may look
like, we know we should have an answer but of course not real number.

Now we also saw that if a and b were both positive then, ab  a b . For now let’s forget
that restriction and do the following.  4  (4)( 1)  4  1

22
Now,  1 is not a real number, but if you think about it we can do this for any square root of a
negative number. For instance,
 16  (16 )( 1)  4  1
 7 (7)( 1)  7  1
etc
So, even if the number is not a perfect square we can still always reduce the square root of a
negative number down to the square root of a positive number (which we or a calculator can deal

with) times  1 . So, if we just had a way to deal with  1 we could actually deal with square
roots of negative numbers. Well the reality is that, at this level, there just is not any way to deal

with  1 so instead of dealing with it we will “make it go away” so to say by using the
following definition.
i  1
Note that if we square both sides of this we get, i 2  1 .
It will be important to remember this later on. This shows that, in some way, i is the only
“number” that we can square and get a negative value.

Using this definition all the square roots above become,


 25  5i
 7 7i
 49  7i
 81  9i etc
Definition: A complex number is a number which can be expressed in the form Z  a  ib where
a and b are real numbers and a is called the real part and ib is called the imaginary part.
So a complex number has two parts the real part and an imaginary part, hence this is a bigger set
than the set of real numbers.
Here are some examples of complex numbers:
1  2i
1
 5i
3
 3  7i
1 i 3
3  2i
These are complex numbers which has a real part and imaginary part
2 , is purely real since the imaginary part is 0, but it can be written as 2  0i
3i , is purely imaginary since the real part is 0, but it can be written as 0  3i
The set of complex numbers contains the set of real numbers, in symbols R  C , that is the set of
real numbers is a proper subset of Complex numbers.

23
If a complex number of the form a  bi , then its conjugate is a  bi , that is, to get a conjugate of
any complex number you simply change the sign of the imaginary part, if it was + change to –
and vice-versa.
Here are some examples of complex numbers and their corresponding conjugates:
Complex number Its Conjugate
1  5i 1  5i
3i 3i
3i  3i
5 5
Notice that the conjugate of a real number is just itself with no changes.
A complex number in standard form is in the form a  bi . Therefore a purely real complex
number is written as a  0i , while a purely imaginary complex number is written in the form
0  bi .

The next topic that we want to discuss here is powers of i. Let’s just take a look at what happens
when we start looking at various powers of i.

Can you see the pattern? All powers if i can be reduced down to one of four possible answers
and they repeat every four powers. This can be a convenient fact to remember.
Stop and think: what is a complex number? Are all real numbers complex, are natural numbers
complex, are rational or irrational numbers complex according to the definition? Don’t forget
the definition of i. lets now move and look at operations on complex numbers.

24
Arithmetic of Complex Numbers
(Add, Subtract, Multiply)

omplex Numbers
ations of adding or subtracting on complex numbers, remember to combine "similar" terms. Also check to see if the answer mus

Addition Rule: (a + bi) + (c + di) = (a + c) + (b + d)i


Add the "real" portions, and add the "imaginary" portions of the complex numbers.
Notice the distributive property at work when adding the imaginary portions.
Additive Identity: (a + bi) + (0 + 0i) = a + bi

Additive Inverse: (a + bi) + (-a - bi) = (0 +0i)

(6 + 4i) + ( 8 - 2i) = (6 + 8) + (4 - 2)i = 14 + 2i

i form.

+ bi form.

25
Subtraction Rule: (a + bi) - (c + di) = (a - c) + (b - d)i
Subtract the "real" portions, and subtract the "imaginary" portions of the complex numbers.
Notice the distributive property at work when subtracting the imaginary portions.

(10 + 3i) - (7 - 4i) = (10 - 7) + (3 - (-4))i = 3 + 7i

26
x Numbers

Multiplying two complex numbers is accomplished in a manner similar to multiplying two binomials.
The distributive multiplication process (sometimes referred to as FOIL) is used.

Distributive Multiplication

Remember that
i 2 = -1

Be sure to replace i2 with (-1).

Multiplication Rule: The product of two complex numbers is a complex number.

The final result is expressed in a + bi form and is a complex number.

Multiplicative Identity: (a + bi) • (1 + 0i) = a + bi

Mutiplicative Inverse:
The number (0 + 0i) has no multiplicative inverse.

27
The conjugate of a complex number a + bi is the complex number a - bi.
For example, the conjugate of 3 + 7i is 3 - 7i.
(Notice that only the sign of the bi term is changed.)

If a complex number is multiplied by its conjugate, the result will be a positive real number
(which, of course, is still a complex number where the b in a + bi is 0).

The product of a complex number and its conjugate is a real number,


and is always positive.

This answer is a real number (no i's).


In addition, since both values are squared, the answer is positive.

(2 + 3i)• (1 + 5i) = 2(1 + 5i) + 3i(1 + 5i) = 2 + 10i + 3i + 15i2


= 2 + 13i + 15(-1) = -13 + 13i

(2 + i)• (2 + i) = 2(2 + i) + i(2 + i) = 4 + 2i + 2i + i2


= 4 + 4i + (-1) = 3 + 4i

(3 - 2i)• (1 - 4i) = 3(1 - 4i) + (-2i)(1 - 4i) = 3 - 12i - 2i + 8i2


= 3 - 14i + 8(-1) = -5 - 14i

28
(3 + 4i)• (3 - 4i) = 3(3 - 4i) + 4i(3 - 4i) = 9 - 12i + 12i - 16i2
= 9 - 16(-1) = 25 (a real number)

with the sign of the imaginary part reversed. It will be noted that

mber.

is facilitated by multiplying both numerator and denominator by giving

tand it may be a bit strange for you to work with complex numbers because of the introduction of i. Now

Binary Operations
In mathematics we are interested in the study of basic operations of algebra, such as:  , , ,  .
You have studied these operations at school, but in this section, you will study the use of these
operations in details.

29
Definition: The operation * is a binary operation in S, if and only if, for every
a , b  S , the a * b  S Otherwise the operation * is not a binary operation in S.
A binary operation on a set A is nothing more than a function from A× A to A. In other words,
it takes pairs of values from A and converts them into single values from A.
You know the product of 3 and 4 as 12. In this case the operation is multiplication, that is *   .
This * can be defined in any way different from the four basic operations given above as will be
illustrated below. Though it may be hard to understand, there is nothing difficult about binary
operations, we just follow how they are defined and then check if the answer is in the given set
of numbers.
Example 1.4.3
(a) State whether the * defined as: a * b  a  b is a binary operation in the set of real numbers.
Solution: To show that * is a binary operation or not, all what is required is to find choices of a
and b were a*b fails. If a=1 and b=5, then a * b  1  5   4 , then  4  R , therefore * is not
a binary operation in the set of real numbers.
(b) State whether * defined as *   , in the set of real numbers. Is * commutative.
Solution: Take any two numbers in the set of real numbers, say
a
 R , hence *   is a binary operation on R. But *   is not
a , b  R , then a * b  a  b 
b
a b
commutative since a * b    b * a we also need to know how to check commutativity, it’s
b a
very simple, just check if a*b = b*a, that’s all. Are you not happy, you thought it will be
difficulty? It’s the simplest. We also need to know how to check associativity, it’s also simple,
just check if a*(b*c) = (a*b)*c. if so then * is associative but if not then * is not associative.
Stop and think: what is a binary operation? Can you show if the operation is commutative,
associative and if it is binary in a given set, if you can, try the following exercise.
Note that these are examination samples, they are easy especially if you do them just now and at
least revise one or two times before an examination, you will really enjoy in an examination.
Exercise 1

Exercise 1.1

1. If A={ 1,2,3,4 } ,B={ 2,4,6,8 } , C ={ 3, 4 , 5, 6 } and the universal set X ={


1,2,3,4,5,6,7,8,9 }.
(a) Find (i) A’ (ii) ( A  B) ' (iii) B – C (iv) ( A  B) ' (v) A  ( B  C )
(vi) ( X  C ) '  ( A  B)
(b) Confirm
(i) The associative laws: ( A  B)  C  A  (B  C ) and
( A  B)  C  A  (B  C )
(ii) the distributive laws: A  ( B  C )  ( A )  ( A  C ) and
A  (B  C )  ( A  B)  ( A  C )
(iii) the De Morgan’s laws: ( A  B) '  A '  B ' and
( A  B)  A  B
' ' '

30
2. (a) If A  B , then simplify if possible (i) ( A  B ) (ii) A '  B ' (iii) A  B '
(iv) A' ( A  B )
3. In each problem below, one of the following relations is true:

A  B, A = B, B  A. Write the correct relation in each case.


A B
a) { x│2x + 3 = 11 – 2x } {x│5x + 4 = x + 12}
b) { x│x2 + 4 = 40 – 6x} {x│4 + 2x = 10}
c) { x│(x – 2)(x – 3) = 0} {x│ x = 2}
d) { x│ x + 4 = 0} {x │x(x + 4) = 0}
e) { x│ x – 1 = 0} U { x│ x – 2 = 0} { x│ x2 – 3x + 2 = 0}
f) { x │x + 3 = 4} { x│(x + 3)2 = 16}
4. Rewrite the following in set builder form:
a) A = [-3, 5) b) B = (3, 8) c) C = [0, 4) d) (-7, -2]
5. Prove that A – B = A ∩ B′
6. Express each of the following in its simplest form
a) [P′ U (Q – P)]′ b) X U (Y U X) c) ( M ∩ N) U ( M ∩ N′) d) A – ( A – B)

7. In the Venn diagram below, shade: (i) W – V (ii) V '  W (iii) V  W '
(iv) V’ – W’

a
8. In problems (a) to (h) find expression of the form for the given decimal expansions, where
b
a and b are integers.
(a) 0.5 (b) 3.12 (c) 11 .34 (d)  4.357 (e) 9.413
(f) 0.9999… (g)1.3333…. (h) 0.85
9. (a) Find an irrational number which lie between (i) 2 and 3 (ii) 19 and 19.01
(iii) -4 and -2
(b) Each of the following operation in I,II and III is a binary operation on R.
I: a*b = (a – b)(a + b) , a, b  R,
II: a*b = ab , a, b  R,
III: a*b = 2 ab , a, b  R
(i) Determine which of I, II III is commutative
(ii) For each of I , II and III , evaluate (3* 2 ) * 5
10. Given the sets X = { 0, 1 } and Y = { 0, 1, 2 }

31
(a) Check whether each of the following operations +, -, x , ÷ is a binary operation on
X and Y.
(b) Also check whether the operation is commutative and associative
11. If A = [ 1, 4] , B = ( 2, 8 ) , C = [ 3, 6 ) and the universal set X = [1, 9 ] . Find each of the
following sets and display it on the number line. (i) B’ (ii) ( A  B) ' (iii) B – C (IV) ( A  B) '
(v) A  ( B  C )
(vi) ( X  C ) '  ( A  B) .

12. An operation * is defined on the set { 3,5,7 } in the table below as follows: a*b is the
result where the row along “a” and column a long “b” meet. e.g. 5*7 = 7.

(i) Is this operation a binary operation on set { 3,5,7 }


(ii) Is this operation commutative
(iii)Evaluate (5*7)*3

FUNCTIONS
Important things to know

 The domain and the range of any given function


 To state whether the function is 1-1 or onto
 To show that the function is 1-1 or not
 To find the inverse of a function
 To write the composite functions in terms of x.
 To state whether the given function is odd, even, or neither.

Relation.

Definition: A relation R from a set A to a set B assigns to each pair (a,b) in AxB exactly one of
the following statements:

(i) “a is related to b”, written aRb


(ii) “ a is not related to b” written

A relation from a set A to the same set A is called a relation in A

Example 2.3.1

32
1. Marriage is a relation from the set M of men to the set W of women. For, given any man
m  M and any w  W , either m is married to w or m is not married to w.
2. Order, symbolized by “<” or equivalently, “x is less than y” is a relation in any set of real
numbers. For, given any order pair (a,b) of real numbers, either a<b or
3. Perpendicularity is a relation in the set of lines in the plane. For, given any pair of lines a
and b , either a is perpendicular to b or a is not perpendicular to b.
4. X = n=set of Natural numbers and Y= n+2

This is a 'notation' for expressing a relation between two variables (say X and Y).

Individual values of these variables are called elements eg Domain; X  1,2,3,4,5,6,.... and Co
domain: . Y  3,4,5,6,7,8,.... Note that Y values are all images of some element in X,
this is also called the Range. The first set of elements (X) is called the domain. The second set
of elements (Y) is called the Co domain. If all elements in Y are some images of some elements
in X the co domain is called the range.

Example 2.3.2 A simple relation like y = x2 can be more accurately expressed using the
 
following format: ( x, y ) : y  x 2 , x, y  R . In this example the Domain is X  x : x  R, that is
all real numbers is the domain. But the co domain is Y  y : y  R , and the Range
Y  y : y  0, y  R

The sketch of the graph of the function y  x 2

Here, you will observe that there are no values of x whose images are negatives. The images are
always 0 or positives numbers. This gives a good distinction example of Co domain and the

33
Range. Y is a set of real numbers but some real numbers are not images of the x values in the
domain.

2.3.1 Inverse Relation

Let R be a relation from A to B. The inverse of R , denoted by R 1 , is the relation from B to A


which consists of those ordered pairs which when reversed belong to R: R 1  (b, a) : (a, b)  R

Example: 2.3.3 Consider a mapping

R  (1,2), (1,3), ( 2,3) R 1  (2,1), (3,1), (3,2)

Example 2.3.4 The inverse of the relations defined by; “ x is the husband of y” and “ x is
taller than y” are respectively. “ y is the wife of x” and “ y is shorter than x”.

2.3.2 One-One mapping

Here one element of the domain is associated with one and only one element of the range.

2.3.3 Onto

A relation is onto if and only if every element in Y is some image of some element in the domain
X. (or if Y is filled)

Example 2.3.1 let a relation be defined as given below:

34
R  (1, a ), ( 2, b), (3, a ), ( 4, b), (5, c )

The co domain is a, b, c and the range is a, b, c , that means the co domain is equal to the
range , hence , the relation is onto.

2.3.4 Many-One mapping

Here more than one element of the domain can be associated with one particular element of the
range.


Example 2.3.5 Consider the relation ( x, y ) : y  x 2 , x, y  Z ,  4  x  4, x  0 . 
Z is the set of integers (positive & negative whole numbers not including zero)

2.3.5 Functions

Notation: y  f (x ) , this notation will be used throughout this book.

Definition: If there is associated with each element of a set X exactly one element of another set
Y, then this association constitutes a function from X to Y and is written f : X  Y .

The unique element in Y assigned to x  X by f denoted by f(x), and called the image of x under
f or the value of f at x. The domain of f is X, and the co domain Y. The range of f is denoted by
f (x ) is the set of images, that is, f ( x )   f ( x ) : x  X . By definition, all functions are relations
but not all relations are functions.

Domain = x1 , x2 , x3 , x4  , the Co domain = y1 , y2 , y3 , y4  , and the range = y1 , y2 , y4  note that y3
is not an image of any element in the domain. Hence by definition f is a function.

35
A function f is into if it is a function by definition, a function f is 1-1 if every element of the
domain X has one and only one image in the co domain Y, a function f is onto if the co domain
is equal to the range.

Example 2.3.6 Let a mapping be

This is not a function because x3 has two images y2 and y4 (refer to the definition of a function).

2.3.5 One to one functions

A function f is 1-1 if every element in the domain X is assigned to exactly one and only one
element in the co domain Y.

Example2.3.7 Let the mapping be given as:

The function is one-to-one, since every element of X has one and only one image in the co
domain Y.

A function f is one to one if and only if for every a , b  X , then , f ( a )  f (b )  a  b . Using this
condition, we can show that any given function is either one to one or not.

Example 2.3.8 Show that the function f ( x )  3 x  1 is one-to-one.

Solution: We take any a , b  X , we show that for any a, b in X f ( a )  f (b )  a  b

If x = a this implies f ( a )  3a  1 and f (b )  3b  1 , then we show f ( a )  f (b )  a  b

That is f (a) = f (b) implies 3a-1 = 3b-1  3a  3b  a  b . Hence the function f ( x )  3 x  1 is


one to one.

36
Example2.3.9 Show that the function f ( x)  3x 2  1 is not one-to-one.

Solution: We take any a , b  X , we show that for any a, b in X f ( a )  f (b )  a  b

If x = a that implies f (a)  3a 2  1 and f (b)  3b 2  1 , then we show f ( a )  f (b )  a  b that


is f (a) = f (b) implies 3a 2  1  3b 2  1 this implies 3a 2  1  3b 2  1
3a 2  3b 2  3a 2  3b 2  0  a 2  b 2  0  (a  b)( a  b)  0  a  b and a  b that we have
a is equal to b and –b. Hence, by definition the function f ( x)  3x 2  1 is not a one to one
function. A property of one-one functions is that on a graph a horizontal line will only cut the
graph once.

2.3.6 Onto functions: A function f is onto if and if the co domain is equal to the range.

Example 2.3.10 Let a mapping be as given:

This relation is a function, by definition, this function is not 1 – 1, but f is onto since the co
domain is equal to the range: Co domain = y1  and the range = y1  .Hence, f is onto.

Example 2.3.11 Let a mapping be

The function f is not onto since the co domain is not equal to the range. In this example the
element y3 is in the co domain but it is not an image of any of the element of the domain.

 
Example 2.3.12 Let a function be defined as: ( x, y ) : y  x  2, x, y  R  , Where R+ is the set of
positive real numbers. Show that the function f is 1-1.

Solution: The domain is all positive real numbers; X  x : x  0, x  R and the range is
Y  y : y  2, y  R. We can use the condition, a function f is one to one if for every
a , b  X , then , f ( a )  f (b )  a  b that is, f ( a )  a  2 and f (b )  b  2 this implies

 
f(a) = f(b) implies a+2= b + 2 That is, a  b , Hence, ( x, y ) : y  x  2, x, y  R  , is one to one.

37
Example 2.3.13 Find the domain and range of the functions given below:

(i) f ( x)  x  1 (ii) f ( x)  x 2  1 (iii) f ( x)  x 2  9

Solutions:

1 0
(i) The expressions not allowed in mathematics are: , ,  ve . So if none of these can
0 0
be arrived at for any real number, then the function is defined (or exist) everywhere. So
you should avoid these expressions. (i) The function f ( x )  x  1 has Domain:
D  x : x  R since there is no value of x which can give you any of the expressions
given above. Range: Range  y : y  R. Note that the domain are the x- values while the
range are the images of x which are the y-values.
(ii) The function f ( x)  x 2  1 has the domain: D  x : x  R, while the range is
Range  y : y  1, y  R . There is no value of x which will give you the y value less
than -1.
(iii) The function f ( x )  x 2  9 , has a radical so, you need to avoid all values of x whose
images will give you negatives. So you take what is in the radical and you want it to be
greater than or equal to zero. So we set, x 2  9  0 these are the only values of x where
x 2  9  x 2  32  0
the function exists. Therefore,
( x  3)( x  3)  0

to satisfy this inequality, you solve this as an equation to find the critical values as:

x = 3 or -3.

The circled signs are the intervals where the function exists. Therefore, the domain:
D  x : x  3, x  R  x : x  3, x  R . The range is: Range  y : y  0, y  R . Note that there
is no value of x which will give an image as negative, and the least image is when x is either 3 or
-3 and the image is 0.

2.3.7 Inverse Function f -1

The inverse function is obtained by interchanging x and y in the function equation and then
rearranging to make x the subject. This technique was used in high schools. Given any function
f ( x )  2 x  3 we make x the subject as follows:
y  2x  3
y  3  2x
y 3
x 38
2
x3
Then the inverse of the given function is f 1 ( x )  . A function f has an inverse if and only if
2
it is one to one. If f -1 exists then, ff 1 ( x)  f 1 f ( x)  x . It is also a condition that the two
functions be 'one to one'. That is the domain of f is identical to the range of its inverse
function f 1 . When graphed, the function and its inverse are reflections either side of the
line y = x.

Example 2.3.14 Find the inverse of the function (below) and graph the function and its inverse
on the same axes.

1
You can graph f(x) and its inverse f ( x) on the same coordinate system as below:

You can observe that the graph of an inverse of a function is simply the mirror image of the
function in the line y  x .

2.3.8 Composite functions

Consider a mapping given below:

39
This is a composite function written as gf , read the composite function of f and g.
gf ( x )  g ( f ( x )) . In this mapping above: gf (3)  g ( f (3))  g (b )  n . Note that X is the
domain of f and Y is the co-domain of f, while Y is the domain of g and Z is the co-domain of g.
But X is the domain of gf and Z is the co-domain of gf . gf  (1, m), (2, n), (3, n), (4, k ) .

Therefore, we can find the inverse of the composite function as;


( gf )  (m,1), (n,2), (n,3), (k ,4) one useful property of inverse composite function is:
1

( gf ) 1  f 1g 1

The composition of two functions g and f is the new function we get by performing f first, and then
performing g. For example, if we let f be the function given by f(x) = x 2 and let g be the function given
by g(x) = x + 3, then the composition of g with f is called gf and is worked out as gf(x) = g(f(x)). So we
write down what f(x) is first, and then we apply g to the whole of f(x). In this case, if we apply g to
something we add 3 to it. So if we apply g to x2, we add three to x2. So we obtain gf(x) = g(f(x)) = g(x 2 ) =
x 2 + 3 . Here is another example of composition of functions. This time let f be the function given by f(x)
= 2x and let g be the function given by g(x) = ex. As before, we write down f(x) first, and then apply g to
the whole of f(x). In this case, f(x) is just 2x. Applying the function g then raises e to the power f(x). So we
obtain gf(x) = g(f(x)) = g(2x) = e 2x . Sometimes the composition of two functions is called a ‘function of a
function’, and sometimes gf is written g◦f.

Order of composition

The order in which we compose functions makes a big difference to the result. You can see this if we
change the order of the functions in the first example. We have taken f(x) = x2 and g(x) = x + 3. Then fg(x)
is given by taking g(x), which is x + 3, and applying f to all of it. This gives us fg(x) = f(x + 3) = (x + 3) 2 = x2 +
6x + 9. You can see that this is not the same as gf(x), because gf(x) = x2 + 3 and this does not in general
equal x2 + 6x + 9.

Decomposition of a function

Sometimes we can write a given function as the composition of two other functions. This is called
decomposing the function. For example, take the function, h(x) = e2x. We have already seen that this
function may be written as a composite function h(x) = gf(x), where g(x) = ex, f(x) = 2x. Let us take
another example. Suppose we have been given the function h(x) = √2𝑥 + 1. Here we see again that the
function can be performed in two stages. We take x, and we first apply the function f(x) = 2x + 1. Then
3 3
we take the cube root of the result. So if we let g(x) = √𝑥 then h(x) = √2𝑥 + 1 = g(2x + 1) = g(f(x)). It is
important to be able to decompose functions in later work in the calculus.

Domains and ranges of composed functions

Sometimes you will meet pairs of functions that cannot be composed. For example, take the two
functions f(x) = −x2 and g(x) = ln(x). Then we know that a square is always positive or zero, so f(x) ≤ 0 for
any x. But we also know that the natural logarithm ln x is defined only for positive numbers. So in this

40
case, g(f(x)) is not defined for any values of x. The composed function gf does not exist for these two
functions g and f. There are also functions that cannot be composed for every x, but that can be
composed if we restrict the values of x. For example, let us take f(x) = 4x − 6, g(x) = √𝑥. Now only
positive numbers, or zero, have real square roots. So g is defined only for numbers greater than or equal
to zero. Therefore g(f(x)) can have a value only if f(x) is greater than or equal to zero. You can work out
3 3
that f(x) ≥ 0 only when x ≥ 2,. So the composed function gf(x) can be defined only for x ≥ 2, and therefore
3
the domain of the function gf is x ≥ 2. In general, the domain of a composed function is either the same
as the domain of the first function, or else lies inside it. If x is a valid input for the composed function gf
then it must be a valid input for the individual function f. The range of a function is the set of all values a
function can take. For example, the range of the function f(x) = ex is given by f(x) > 0, because e x is
always greater than zero. As another example, if f(x) = sin x then the range is given by −1 ≤ f(x) ≤ 1. If we
have a composed function gf then its range must lie within the range of the second function g. Here is an
example to show this. Take f(x) = x − 8, g(x) = x2.

Now the first function f(x) can take any value. But the second function g(x) sends any number to its
square, which is greater than or equal to zero. So gf(x) = g(x − 8) = (x − 8)2 = x2 − 16x + 64 ≥ 0 . Whatever
f(x) is, g(f(x)) must be greater than or equal to zero, because g applied to anything is greater than or
equal to zero. So the composed function gf(x) = x2 − 16x + 64 can take only values that are greater than
or equal to zero. What happens if we compose the functions the other way round? We shall take g first,
and then f. So f(g(x)) = f(x2 ) = x2 − 8 ≥ −8 . The range of fg is given by f(g(x)) ≥ −8. So in this case, the
range of the composed function f(g(x)) is contained in the range of f, but it is not the whole of the range
of f. And in general, the range of a composed function is either the same as the range of the second
function, or else lies inside it. If a value is a possible output from a composed function then it must be a
possible output from the second function

Example 2.3.15 Let f ( x )  3 x  1 and g ( x )  x  1 , express in terms of x the following:

(i) g f , (ii) ( g  f ) 1 (iii) gof ( 2)

Solutions

(i) gf ( x)  g ( f ( x))  g (3 x  1)  (3 x  1)  1  3 x


(ii) Here you need to find the inverse of each function as follows:

f ( x)  3x  1
y  3x  1 x 1
Therefore, f 1 ( x ) 
3x  y  1 3
y 1
y
3

g ( x)  x 1
y x 1 Therefore, g 1 ( x)  x 2  1
y2  x 1
x  y2  1

41
Therefore, using the condition, ( gf ) 1  f 1g 1
x2  1  1 x2 x2
( gf )  f g  f ( g )  f ( x  1) 
1 1 1 1 1 1
 Therefore, ( gf ) 
2 1
You
3 3 3
can verify this by finding the inverse of gf using the conventional method.

gof ( x )  3 x by (i) above , gof ( 2)  3( 2)  6


(iii)

Exercises

For the following pairs of functions, find the domain and range of the composed function gf: (a) f(x) = 2x,
1
g(x) = sin x, (b) f(x) = x2, g(x) = ex (c) f(x) = −x, g(x) = ln x, (d) f(x) = 𝑥, g(x) = 2 sin x.

Answers

(a) domain is all real x, range is −1 ≤ y ≤ 1 (b) domain is all real x, range is y ≥ 1 (c) domain is x < 0, range
is all real y (d) domain is all real except x= 0, range is −2 ≤ y ≤ 2

2.3.9 Even and odd functions

Definition: A function f is an even function if and only if f (  x )  f ( x ) , and a function is odd if


and only if f (  x )   f ( x ) . In establishing on whether a function is odd or even, all you need is
to verify these two conditions. If both conditions fail, then a function is neither odd nor even. In
fact there are some functions which are neither even nor odd.

Example 2.3.16 Verify whether each of the given function is even, odd or neither.

(i) f ( x)  x 4  2 x 2 (ii) f ( x)  x 3  2 x (iii) f ( x)  2 x 2  1

Solutions

(i) Using the conditions given above, you proceed as follows: Let f ( x)  x 4  2 x 2 , and
f ( x)  ( x) 4  2( x) 2  f ( x)  x 4  2 x 2  f ( x) , hence f (  x )  f ( x ) is satisfied.
That means the function f ( x)  x 4  2 x 2 , is an even function.
(ii) Let f ( x)  x 3  2 x , and f ( x)  ( x)3  2( x)   x 3  2 x  [ x 3  2 x]   f ( x) , that
means the function f ( x)  x  2 x , is an odd function.
3

(iii) Let f ( x)  2 x 2  1 , and f ( x)  2( x) 2  1  2 x 2  1   f ( x) or  f ( x) , hence


f ( x)  2 x 2  1 is neither even nor odd,

If a function is not even and not odd then it is said to be neither.

42
By now you must be happy when you hear about functions, about their domains, range and
even to sketch. You must be able to give a definition of old and even functions, they are very
important. Make sure you have understood before you attempt to answer the next exercise.

Try the following questions to check your understanding.

Exercise 2

1
1. The function f and g are defined by f ( x)  x 2 and g ( x )  x  1 for all real values of x in the
2
domain 0  x  3 .

(a) Find f 1 ( x) and g 1 ( x) in terms of x


(b) Sketch , in the same diagram, the graphs of f, g, f 1 ( x) and g 1 ( x)

1
2. The functions f and g are defined by f ( x )  2 x  1, x  R and g ( x )  , x  R, x  0
x

(a) Calculate the value of gf(2)


(b) Find g 1 ( x) in terms of x
(c) Calculate the value of x for which fg(x) = x

25
3. The functions f and g are defined by f ( x)  , x  R ,1  x  9 and
3x  2
f ( x)  x 2 , x  R, 1  x  3 . Find

(a) The range of f


(b) The inverse function, f 1 ( x) , state its domain
(c) The composite function fg and state its domain
2
(d) The solution of the equation fg ( x ) 
x 1

4. Given the function f ( x )  1  x

(i) Find the domain of f(x)


(ii) Find the range of f(x)
(iii) Sketch the graph of f(x)
5. Given the function g(x) = – | x – 3 |
(i) Find the domain of g(x)
(ii) Find the range of g(x)
(iii) Sketch the graph of g(x)
(iv) Find the intersection of g(x) with the line 2y + x = – 2

43
6. The fish population y in a certain river is related to the tons of pollutants x according to
the following y = 100,000 – 1 500x
(a) What are the domain and the range of this function?
(b) Sketch the graph of this function.
(c) Suppose that 17.5 tons of pollutants is deemed critical. How many fish would
be in the stream at the critical level of pollution?
7. (a) Show that the following functions are not 1 – 1 .
(i) y = x2 + 3x + 2 (ii) y = x2
(b) Show that the following functions are 1 – 1.
(i) y = 3x + 4 (ii) y = 5
8. For each of the following functions state whether the function is odd, even or neither of
these. (i) f(x) = 4 – 3x2 (ii) f(x) = 3x2 + x
1
(iii) f ( x )  x  (vi) f(x) = 2 + x2
x

Linear and quadratic functions


The simplest function defined in mathematics by means of a non-trivial algebraic expression is
the function defined by the equation y  f ( x )  mx  c.
Definition: The function f , defined by the first degree equation f  ( x, y ) : y  mx  c, where
m and c are constants, is called a linear function.
Example 3.3.1 Solve the following equations 4 x  5  x  7
Solution:
4x  5  x  7
4x  x  7  5
3x  2
2
x
3
Example 3.3.2 Solve the equation x  1  3
Solution: This equation has solution x  4 . But if you square both sides you will get: ( x  1) 2  9 .
If this equation is solved, one of its roots i.e. -2, although it does not satisfy the original
equation. Note that when an expression is squared more roots are introduced in the solution set.
Accepted solutions which satisfies the original equation.
2x  5 5x
Example 3.3.3 Solve the equation  x
2 x 1
Solution:

44
2x  5 5x
 x
2 x 1
( 2 x  5)( x  1)  5 x ( 2)
x
2( x  1)
2x2  5x  2x  5  2x2  2x
x  1

3.4 The theory of quadratic equations (Maximum and Minimum)


The second type of algebraic function usually considered is one in which the defining equation is
of the second degree.

Definition: The function f , defined by the second degree equation f  ( x, y ) : y  ax 2  bx  c , 
is called a quadratic function. Note that y  f ( x)  ax  bx  c is a quadratic function while
2

ax 2  bx  c  0 is called a quadratic equation. In general, the equation has solutions called zeros
of a function or the roots of the equation f ( x )  0 .
A quadratic function has a turning point which is either the maximum point or a minimum point.
The maximum or minimum point of a quadratic function can be determined by the coefficient of
the x 2 . If a  0 , the function has a minimum, but if a  0 , the function has a maximum.
The graph of the general quadratic function may be sketched by the more direct process of
expressing the quadratic function in terms of the square of a linear function. Consider the
function f ( x)  ax 2  bx  c , by completing squares of this function as follows:
f ( x)  ax 2  bx  c factor out a
b c
f ( x)  a( x 2  x  ) , take the coefficient of x , half it and square it and add it as a
a a
zero.
b b b c
f ( x)  a( x 2  x  ( )2  ( )2  )
a 2a 2a a
 b 2 b 2
c
f ( x )  a ( x  )  2  
 2a 4a a
 b 2 4 ac  b 2 
f ( x )  a ( x  )  
 2a 4a 2 

b
Since the square quantity ( x  )  0 , the expression within the bracket has its least value when
2a
b b
x . If a  0 , the function has its least value at x   . This least value of the function,
2a 2a
4ac  b 2 b 4ac  b 2
( ) , is called its minimum. The least point, (  , ) , is called the minimum
4a 2a 4a
b
point. If, however, a  0 , when x   , the function has its greatest value called its maximum,
2a
4ac  b 2 b 4ac  b 2
and is also equal to ( ) . The greatest point, (  , ) , is called the maximum
4a 2a 4a

45
b 4ac  b 2
point. In either case the point, ( , ) , is called the vertex of the parabola, with this
2a 4a
point found, and an additional point or two points , the graph can be sketched like the one
below:

Tip: maximum and minimum points are very important in sketching graphs, make sure
you understand how to get these points.

3.5 Nature of the roots of quadratic equations

The general form of a quadratic equation is: ax 2  bx  c  0 , where a, b & c are constants

The expression b 2  4 ac is called the discriminate and it is given the letter Δ (delta).

All quadratic equations have two roots/solutions. These roots are real (Distinct or equal) or
complex. *complex - involving the square root of -1

You can determine the nature of the roots of a given quadratic equation by calculating the value
of the discriminant b 2  4 ac   . If

(i) b 2  4 ac  0 , then the quadratic equation has two distinct real roots. That is   0
(ii) b 2  4 ac  0 , then the quadratic equation has one real root.

46
(iii) b 2  4 ac  0 , the quadratic equation has imaginary root

These three properties are very important, always examinable and if well understood, they are
the easiest one would always want to use in any question that involves the discriminant.

The nature of the roots can be used to determine whether the solution of the given quadratic
equation has one root for (ii), two distinct roots for (i) and imaginary roots for (iii).

Example: Determine the nature of the roots of the following quadratic equations

(i) 6  x  x2  0

Solution; This equation ha a = -1, b =-1 and c = 6, the discriminant is b 2  4 ac . On


substitution b 2  4ac  (1) 2  4(1)( 6)  1  24  25  0

That means the equation has two distinct roots.

(ii) 2 x 2  3x  5  0

Solution; This equation ha a = 2, b =-3 and c = 5, the discriminant is b 2  4 ac . On


substitution b 2  4ac  (3) 2  4(2)(5)  9  40  31  0

That means the equation has imaginary roots.

Solution of quadratic equation

You are now in a position to find the zeros of quadratic equation. Recall that the equation
f ( x )  0 , which is the values of x where y  0 , are what you are interested. Generally, two
values of x can be found, but these values can be determined by the discriminate b 2  4 ac
without solving the equation. The zeros or the solutions of a quadratic equation are values of x
where the graph f ( x)  ax  bx  c crosses or meets the x-axis. The two roots can be found
2

graphically, by factorization, by using a formula, or by completing the squares. In this section a


method of completing squares will be demonstrated below.

Example 3.6.1 Solve the following equation ax  bx  c  0 , using the method of completing
2

squares.

Solution: Take the equation

47
ax 2  bx  c  0 transpose c
ax 2  bx   c Divide by a through out
b c
x2  x take the coefficien t of x , half it and square it and add it to both sides
a a
2 2
b  b   b  c
x  x
2
   
a  2a   2a  a
2
 b  b2 c
x   
 2a  2
4a a
2
 b  b 2  4 ac
x   take square root both sides
 2a  4a 2
b b 2  4 ac
x 
2a 4a 2
b b 2  4 ac
x  solve for x
2a 2a
b b 2  4 ac
x  to give you the formula
2a 2a
b b 2  4 ac
x
2a
 b  b 2  4ac  b  b 2  4ac
There are two roots or solutions such as x  or x  . These are
2a 2a
the roots, or zeros, or solutions of the quadratic equation ax 2  bx  c  0 in solving quadratic
equation, you can use any method provided the method of solving is not given in the question.

Example 3.6.2 Solve the equation 3 x  x  10 , using completing square method.


2

Solution: you set

48
3 x 2  x  10
1 10
x2  x 
3 3
2 2
1  1 10  1
x2  x       
3  6 3  6
2
 1 10 1
x    
 6 3 36
2
 1 120  1
x    take square root both sides
 6 36
1 121
x   
6 36
1 11
x    solve for x
6 6
1 11
x  
6 6
5
x  2 or
3

5
The two roots are 2 or . If the method of solving is not specified in the question, any method
3
may be used and it will give the same solutions.

Example 3.6.3 Find the two values of x that satisfy the following quadratic equation:

In this equation, you have;

49
Relations between zeros and coefficients of the quadratic equations
If  and  are roots of a given quadratic equation, then the standard form of a quadratic
equation is;

x 2  (   ) x    0 , and the general equation of a quadratic equation is ax 2  bx  c  0 , you


b c
divide this later equation by a throughout, you get: x 2  x   0 . On comparing the two
a a
equations, you will get:

b c
  and  
a a

Example 3.6.4 without solving, form a quadratic equation whose roots are the squares of the
roots of the equation: 2 x 2  x  6  0

Solution: Let  and  be the roots of the given equation 2 x 2  x  6  0 . You compare the
equation x 2  (   ) x    0 , with the given equation with 1 as the coefficient of x 2 ,
1 1
x2  x3 0 and then you will have,      , and   3 . Since
2 2
25
(   ) 2   2  2   2 , the sum of the roots for the new equation is,  2   2  , and the
4
25
product is, ( ) 2  9 . Therefore, the required equation is x 2  x  9  0 , which is
4
4 x 2  25 x  36  0 .

Stop and think: do you now know the quadratic formula, the relationship between the roots of a
quadratic equation, if so, try the following problems and make sure you get the answers given
below.

PROBLEMS

1. Given that the equation kx2 + 12x + k = 0, where k is a positive constant, has equal
roots, find the value of k.
2. The equation 2x2 − 3x − (k + 1) = 0, where k is a constant, has no real roots. Find the set of
possible values of k.
3. The equation x2 + kx + (k + 3) = 0, where k is a constant, has different real roots. Find the set of
possible values of k.
4. The equation x2 + kx + 8 = k has no real solutions for x. find the set of possible values of k.
Solutions
1. k = 6
2.
3. k < −2, k > 6
4. −8 < k < 4

50
APPLICATIONS OF QUADRATIC
If 1200 cm2 of material is available to make a box with a square base and an open top, find
the largest possible volume of the box.
Solution. Let s be the length of the side of the square base; let h be the height of the box; let
V be the volume of the box. Then V = hs2. We want to maximize V , so we need to write h in
terms of s (or s in terms of h).
The surface area of the box is

(area of base) + 4 · (area of one side) = s2 + 4hs,

so 1200 = s2 + 4hs. We can solve this equation for h in terms of s:

Plugging this into the expression for V , we get . Therefore,


,
and this is V as a function of one variable, s. What is its domain? The volume cannot√

be negative, which means 300 0; solving this inequality for s yields s ≤ −20 3

√√
or 0 ≤ s ≤ 20 3. But s ≥ 0 rules out the possibility of s ≤ −20 3, so the condition is

√h √i
0 ≤ s ≤ 20 3. Therefore, the domain is 0,20 3 . (There are other ways of getting the righthand
endpoint of this domain, or other reasonable choices of righthand endpoint.)
Now we need to find the absolute maximum of on this domain. The
derivative is , and this is never undefined, so we can find the critical numbers by
setting it to zero:

,
but −20 is not in the domain, so the only critical number is s = 20.
Now we can use the “closed-interval method” to determine the absolute maximum of V . We
test the critical number and the endpoints of the domain:

V (0) = 0; = 4000; .

51
The highest of these is V (20) = 4000, so the absolute maximum is 4000. Therefore, the largest
possible volume is 4000 cm3. (This is achieved when the base side length is s = 20 cm

and the height is = 10 cm.)

Polynomials
Introduction

A functions of the form f ( x )  ao x n  a1 x n 1  a2 x n  2  ....  an 1 x  an , where ai , for i  1,2,3,..., n


constants and n is a positive integer. This is called a polynomial function of order n. If you set
n 1 n2
f ( x )  0 , then we have ao x  a1 x  a2 x  ....  an 1 x  an  0 , this called a polynomial
n

equation of order n. If you have a function of the form: f ( x)  3x 5  2 x 3  x  1 this is a


polynomial of order 5. And if you have an equation of the form 3 x 4  2 x 3  x  1  0 this is a
polynomial equation of order 4. Note that a polynomial of order n has a finite sum of n terms.
Our task on polynomials is either to sketch if you have a polynomial function or solving for the
polynomials roots if you have a polynomial equation. To do this we shall begin by looking at
polynomial expression called algebraic expressions. We should look at how to divide expression
by another expression.

3.7.2 Division of algebraic expressions

Just as you divide any real number by another real, such as 345 divided by 34, you carried out
the operations as follows: 34 342 this can also be done when dividing algebraic expressions.

Long division: If f(x) is divided by d(x) and d(x) is non-zero, and the degree of d(x)  the
degree of f(x), then two unique polynomials q(x) the quotient and r(x) the remainder exist, this is
called improper rational expression , so that:

f ( x) r ( x)
 q( x) 
d ( x) d ( x)

Note - the degree of r(x) is less than the degree of d(x), and this called proper rational expression.

52
5 x 3  x 2  3x  2
Example 3.7.1 Find the quotient and remainder when
x2  2x  5

6 x  53
Therefore, q ( x )  5 x  11 is the quotient and r ( x )   is the remainder.
x  2x  5
2

3.7.3 The Remainder Theorem: If a polynomial f(x) is divided by (x-a), where a is any
constant, until a constant remainder independent of x is obtained, this remainder is equal to

f (a).

Example 3.7.2 Find the remainder when (2x3+3x+x) is divided by (x+4).

The reader may wish to verify this answer by using algebraic division.

3.7.4 The Factor Theorem. (a special case of the Remainder Theorem) . If f (a) = R is zero,
then, a is a zero of f(x), then (x−a) is a factor of the polynomial f(x).

The converse is true, that is, If (x-a) is a factor of f(x), then f(a) = R = 0, and a is a zero of
f(x).

53
Example 3.7.3

Synthetic division: You can determine the quotient and remainder when an expression f(x) is
divided by a factor x-a, where a is a constant using a method called synthetic division.

The process of division for polynomial in x (or any one letter) may be greatly simplified when
the divisor is in the form x – a. This process, known as synthetic division, will be illustrated.

Example 3.7.2 Divide 5 x 3  14 x  3 by x – 2 using synthetic division.

Solution: You write the coefficients of the expression as follows:

The procedure is as follows: after writing the coefficients of the expression, you find a, in this
case a = 2. You write the division upside down, drop 5 the coefficient of the highest power,
multiply this by a, in this case by 2, write the answer under the next coefficient, add write the
answer below, in this case 10, multiply your answer by 2, write the answer, 20 under the next
coefficient, add, your answer is 6, multiply 6 by 2 and write your answer under the constant 3,

54
add, you will get 15. The number 15 is the remainder, while your quotient is: 5 x 210 x  6 and
remainder 6. This is easier and faster in find the quotient and remainder, than the long division.

Fundamental Theorem of algebra: Every polynomial function whose defining equation is


f ( x )  ao x n  a1 x n 1  a2 x n  2  ....  an 1 x  an

For n  1 and a0  0 , has at least one (real or complex) root or at most n roots.

Stop and think: what is a polynomial, what are the zeros of a polynomial, can you perform a
long division given two polynomial, and are you able to perform synthetic division? Have you
understood the Remainder theorem/ Factor theorem? If so follow nicely the way question one
has been done for you and then try the rest.

EXERCISE
1. Factorise the expression x3−17x2+54x−8 given that one of the factors is (x−4).

Solution
Given that x −4 is a factor we can write x3 −17x2 +54x −8 = (x −4)× (a

quadratic polynomial)

The polynomial must be quadratic because the expression on the left is cubic and x −4 is linear. Suppose
we write this quadratic as ax2 + bx + c where a, b and c are unknown numbers which we need to find. Then

x3 −17x2 +54x −8 = (x −4)(ax2 + bx + c)


Removing the brackets on the right and collecting like terms together we have
x3 −17x2 +54x −8 = ax3 +(b −4a)x2 +(c −4b)x −4c

55
Solution (contd.)
Like terms are those which involve the same power of the variable (x).
Equating coefficients means that we compare the coefficients of each term on the left with the
corresponding term on the right. Thus if we look at the x3 terms on each side we see that x3 = ax3 which
implies a must equal 1. Similarly by equating coefficients of x2 we find −17 = b−4a With a = 1 we have −17
= b − 4 so b must equal −13. Finally, equating constant terms we find −8 = −4c so that c = 2.

As a check we look at the coefficient of x to ensure it is the same on both sides. Now that we know a = 1,b
= −13,c = 2 we can write the polynomial expression as x3 −17x2 +54x −8 = (x −4)(x2 −13x +2)

2. Let the polynomial expression x3 −17x2 +54x −18 be denoted by P(x). Verify that x = 4 is a
solution of the equation P(x) = 0. Hence find the other solutions.

56
Solution
We substitute x = 4 into the polynomial expression P(x):
P(4) = 43 −17(42)+54(4)−8 = 64−272+216−8 = 0
So, when x = 4 the left-hand side equals zero. Hence x = 4 is indeed a solution. Knowing that x = 4 is a root
we can state that (x−4) must be a factor of P(x). Therefore P(x) can be re-written as a product of a linear
and a quadratic term:
P(x) = x3 −17x2 +54x −8 = (x −4)× (quadratic polynomial)
The quadratic polynomial has already been found in a previous task so we deduce that the given equation
can be written

P(x) = x3 −17x2 +54x −8 = (x −4)(x2 −13x +2) = 0 In this form we

see that x −4 = 0 or x2 −13x +2 = 0

The first equation gives x = 4 which we already knew.


The second equation must be solved using one of the methods for solving quadratic equations given in
Section 3.2. For example, using the formula we find

− b ± b2 − 4ac
x = a =1 ,b = − 13,c =2
2a
13 ± p ( − 13)2 − 4. 1. 2
=
√ 2
13 ± 161 13 ± 12. 6886
= =
2 2

So x = 12.8443 and x = 0.1557 are roots of x2 −13x +2.


Hence the three solutions of P(x) = 0 are x = 4, x = 12.8443 and x = 0.1557, to 4 d.p.

3. Plot a graph of the function y = 4x4 −15x2 +5x +6 and hence approximately solve the
equation 4x4 −15x2 +5x +6 = 0.

57
y

x
−5 5

y = 4 x 4 − 15x 2 +5 x +6

x = 1 x = 1 . 5 x = − 0. 5 x = − 2

An important feature of the graph of a polynomial is that it is continuous. There are never any gaps
or jumps in the curve. Polynomial curves never turn back on themselves in the horizontal direction,
(unlike a circle). By studying the graph in Figure 6 you will see that if we choose any two values of x,
say a and b, such that y(a) and y(b) have opposite signs, then at least one root lies between x = a
and x = b.

PROBLEMS
1. Factorise x3 − x2 −65x −63 given that (x +7) is a factor.

2. Show that x = −1 is a root of x3+11x2+31x+21 = 0 and locate the other roots algebraically.

3. Show that x = 2 is a root of x3 −3x −2 = 0 and locate the other roots.

4. Solve the equation x4 −2x2 +1 = 0.

5. Factorise x4 −7x3 +3x2 +31x +20 given that (x +1) is a factor.

6. Given that two of the roots of x4 +3x3 −7x2 −27x −18 = 0 have the same modulus but different
sign, solve the equation.

(Hint - let two of the roots be α and −α and use the technique of equating coefficients).

58
ANSWERS

Answers

1. (x +7)(x +1)(x −9)


2. x = −1,−3,−7
3. x = 2,−1 (repeated)
4. x = −1,1 (each root repeated)
5. (x +1)2(x −4)(x −5)
6. (x +3)(x −3)(x +1)(x +2)

Equations with Radicals


The title seems to imply that we’re going to look at equations that involve radicals. However,
we are going to restrict ourselves to equations involving square roots. The techniques involved
here can be used to solve equations with other radicals, however the work is usually significantly
messier than when dealing with square roots. So in this section, we will work with square roots
only.

Before proceeding it should be mentioned that topic that include equations reducible to quadratic
form will be discussed separately. The reason is that we will in fact end up solving a quadratic
equation in most cases. However, the approach is significantly different and so we’re going to
separate the two topics into different sections in this book.
Example 1 Solve x  x  6 x  0
Solution
Here the basic problem is how to remove the square root. If there was no square root, we
would solve the equation very easily. The first thing that we need to do is to remove the
square root. To do that we transpose the term with a square root to the right, as follows. If
you square the equation both sides as it is, the square root will not go. This is how it is
done;

59
x x6 0
x x6
( x) 2  ( x  6 ) 2
x2  x  6
x2  x  6  0
( x  3)( x  2)  0
x  3 or x  2
Not that squaring both side produces a quadratic equation which when factorized two values x =
3 and x = -2 are found as solution to a quadratic equation and not radical equation. Find the
solution by substituting these values in order to find the value which satisfies the original
equation.
That is if x = 3, the equation become,
x x6
3  36
3 9
33
Which is true.
That is if x = -2, the equation become,

x x6
2  26
2 4
22
Which is false.

In this case the solution is x = 3. Note that x = -2 is not one of the solution as it does not satisfies
the original equation.
Note that when squaring both sides of the equation the original equation is changed and in the
process introduced a solution that is not a solution to the original equation. With these problems
it is very important that the solutions that satisfies the original equation. Take the values that are
actual solutions to the original equation. Care must be taken when solving such equations. When
solving the quadratic equation two solutions are generally found and it is possible both of these,
or one of these, or none of these values will be the solution(s) to the original equation. The
only way to know is to check your solutions!

Example 2 Solve each of the following equations


(a) y  y  4  4
Solution
If you square both sides you are likely to encounter problems. Before discussing the problem
we’ve got here let’s make sure you can do the squaring that we did above since it will show up

60
on occasion. All that we did here was use the formula (a  b) 2  a 2  2ab  b 2 ,
( y  y  4 ) 2  (4) 2 Implies a  y and b  y  4
When we square both sides the there is still a square root in the problem and we’ve made the
remainder of the problem messier as well. So, what we’re going to need to do here is make sure
that we’ve got a square root all by itself on one side of the equation before squaring. Once that is
done we can square both sides and the square root really will disappear. The easiest way is to
transpose y to the right as follows and square both sides.
y y4 4
y4  4 y
( y  4 ) 2  (4  y ) 2
y  4  16  8 y  y 2
y 2  9 y  20  0
( y  5)( y  4)  0
y  5 or y  4
When checking which solution satisfies the origin equation, we see that;
For x = 5, we have
y y4 4
5 54  4
53 4
8 is not equal to 4, Therefore, x = 5 is not a solution.
For y = 4, we have
y y4 4
4 44  4
44
Which is true. Therefore, x = 4, is the solution to the original equation.

(b) 1  x  2x  3

Solution. 1  x  2x  3
Now we can solve this problem fast as follows.
Transpose x to the left and square both sides

61
1  x  2x  3
1 x  2x  3
(1  x ) 2  ( 2 x  3 ) 2
1  2x  x 2  2x  3
x 2  4x  4  0
( x  2)( x  2)  0
x2
If x = 2, then
1  x  2x  3
1 2 43
1  2 1
1 3
Which is false.
So, x = 2 isn’t a solution to the original equation. Since there is only one possible solution, this
means that there are no solutions to the original equation. This doesn’t happen too often, but it
does happen at times.

(a) 4z  3  1  z

Solution:
Transpose 1 to the right as follows and square both sides;
5z  6  2  z
5z  6  2  z
( 5 z  6 ) 2  ( z  2) 2
5z  6  z 2  4z  4
z2  z  2  0
( z  2)( z  1)  0
x  2 or x  1
Checking the solutions to this quadratic equations, we have;
If z = 2, we have
5z  6  2  z
10  6  2  2
42 2
22
Which is true, so x = 2 is a solution to the original equation.
If z = -1, we have

62
5z  6  2  z
56 2  2
1 2  2
1  2
Which is false, therefore, z = -1 is not a solution to the original equation.

The examples given above has shown that once we get our list of possible solutions anywhere
from none to all of them can be solutions to the original equation. Always remember to check
your answers!

So far, all the equations that we’ve looked at have had a single square root in them. Now we
look at problems with more than one square root in the equations. In the following examples
with more than one square root, we need to get a square root all by itself on one side of the
equation before squaring. Once that is done we can square both sides and the square root really
will disappear.
So, when there is more than one square root in the problem we need to check whether our
possible solutions satisfies the original equation. It is possible that anywhere from none to all of
the possible solutions will in fact be solutions and the only way to know for sure is to check them
in the original equation.

Example 3. Solve each of the following


(a) 2 x  1  x  4  2
(b) x  7  2  3  x
Solutions
(a) 2 x  1  x  4  2
Before squaring both sides, make sure that we do not have two square roots on one side
of the equation. If you square both sides with two square roots on one side, you will
complicate the problem.
To this problem, we transpose one square root to the rigxht so that we have one square
root on each side, and square both sides. So we have the following;
2x  1  x  4  2
2x  1  x42
( 2 x  1) 2  ( x  4  2) 2
2x  1  x  3  4  4 x  4
x24 x4
( x  2) 2  ( 4 x  4 ) 2
x 2  4 x  4  16 ( x  4)
x 2  20 x  68  0
(b) x 7  2  3 x

63
x 7  2  3 x
( x  7  2) 2  ( 3  x ) 2
x 7 4 4 x7  3 x
x  11  4 x  7  3  x
4 x  7  8  2 x
( 4 x  7 ) 2  ( 8  2 x ) 2
16 ( x  7 )  64  4 x 2  32 x
16 x  112  64  4 x 2  32 x
4 x 2  16 x  64  112  0
4 x 2  16 x  48  0

PARTIAL FRACTIONS
Partial-fraction decomposition is the process of starting with the simplified answer and taking it
back apart, of "decomposing" the final expression into its initial polynomial fractions
There are several methods for determining the coefficients for each term and we will go over
each of those as we work the examples. Speaking of which, let’s get started on some examples.

Example 1 Determine the partial fraction decomposition of each of the following.

(a)

(b)

(c)

(d)
Solution
We’ll go through the first one in great detail to show the complete partial fraction process and
then we’ll leave most of the explanation out of the remaining parts.

(a)
The first thing to do is factor the denominator as much as we can.

64
So, by comparing to the table above it looks like the partial fraction decomposition must look
like,

Note that we’ve got different coefficients for each term since there is no reason to think that they
will be the same.

Now, we need to determine the values of A and B. The first step is to actually add the two terms
back up. This is usually simpler than it might appear to be. Recall that we first need the least
common denominator, but we’ve already got that from the original rational expression. In this
case it is,

Now, just look at each term and compare the denominator to the LCD. Multiply the numerator
and denominator by whatever is missing then add. In this case this gives,

We need values of A and B so that the numerator of the expression on the left is the same as the
numerator of the term on the right. Or,

This needs to be true regardless of the x that we plug into this equation. As noted above there are
several ways to do this. One way will always work, but can be messy and will often require
knowledge that we don’t have yet. The other way will not always work, but when it does it will
greatly reduce the amount of work required.

In this set of examples the second (and easier) method will always work so we’ll be using that
here. Here we are going to make use of the fact that this equation must be true regardless of

65
the x that we plug in.

So let’s pick an x, plug it in and see what happens. For no apparent reason let’s try plugging
in . Doing this gives,

Can you see why we choose this number? By choosing we got the term
involving A to drop out and we were left with a simple equation that we can solve for B.

Now, we could also choose for exactly the same reason. Here is what
happens if we use this value of x.

So, by correctly picking x we were able to quickly and easily get the values of A and B. So, all
that we need to do at this point is plug them in to finish the problem. Here is the partial fraction
decomposition for this part.

Notice that we moved the minus sign on the second term down to make the addition a
subtraction. We will always do that.

(b)

66
Okay, in this case we won’t put quite as much detail into the problem. We’ll first factor the
denominator and then get the form of the partial fraction decomposition.

In this case the LCD is and so adding the two


terms back up give,

Next we need to set the two numerators equal.

Now all that we need to do is correctly pick values of x that will make one of the terms zero and
solve for the constants. Note that in this case we will need to make one of them a fraction. This
is fairly common so don’t get excited about it. Here is this work.

The partial fraction decomposition for this expression is,

67
(c)
In this case the denominator has already been factored for us. Notice as well that we’ve now got
a linear factor to a power. So, recall from our table that this means we will get 2 terms in the
partial fraction decomposition from this factor. Here is the form of the partial fraction
decomposition for this expression.

Now, remember that the LCD is just the denominator of the original expression so in this case

we’ve got . Adding the three terms back up


gives us,

Remember that we just need to add in the factors that are missing to each term.

Now set the numerators equal.

In this case we’ve got a slightly different situation from the previous two parts. Let’s start by
picking a couple of values of x and seeing what we get since there are two that should jump right
out at us as being particularly useful.

68
So, we can get A and C in the same manner that we’ve been using to this point. However, there
is no value of x that will allow us to eliminate the first and third term leaving only the middle
term that we can use to solve for B. While this may appear to be a problem it actually isn’t. At
this point we know two of the three constants. So all we need to do is chose any other value
of x that would be easy to work with ( seems particularly useful here), plug that
in along with the values of A and C and we’ll get a simple equation that we can solve for B.

Here is that work.

In this case we got this will happen on occasion, but do not expect it to
happen in all cases. Here is the partial fraction decomposition for this part.

(d)
Again, the denominator has already been factored for us. In this case the form of the partial
fraction decomposition is,

69
Adding the two terms together gives,

Notice that in this case the second term already had the LCD under it and so we didn’t need to
add anything in that time.

Setting the numerators equal gives,

Now, again, we can get B for free by picking .

To find A we will do the same thing that we did in the previous part. We’ll use
and the fact that we know what B is.

In this case, notice that the constant in the numerator of the first isn’t zero as it was in the
previous part. Here is the partial fraction decomposition for this part.

70
Now, we need to do a set of examples with quadratic factors. Note however, that this is where
the work often gets fairly messy and in fact we haven’t covered the material yet that will allow
us to work many of these problems. We can work some simple examples however, so let’s do
that.

Example 2 Determine the partial fraction decomposition of each of the following.

(a)

(b)

Solution

(a)
In this case the x that sits in the front is a linear term since we can write it as,

and so the form of the partial fraction decomposition is,

Now we’ll use the fact that the LCD is and add
the two terms together,

71
Next, set the numerators equal.

This is where the process changes from the previous set of examples. We could choose
to get the value of A, but that’s the only constant that we could get using this method
and so it just won’t work all that well here.

What we need to do here is multiply the right side out and then collect all the like terms as
follows,

Now, we need to choose A, B, and C so that these two are equal. That means that the coefficient
of the x2 term on the right side will have to be 8 since that is the coefficient of the x2 term on the
left side. Likewise, the coefficient of the x term on the right side must be zero since there isn’t
an x term on the left side. Finally the constant term on the right side must be -12 since that is the
constant on the left side.

We generally call this setting coefficients equal and we’ll write down the following equations.

Now, we haven’t talked about how to solve systems of equations yet, but this is one that we can
do without that knowledge. We can solve the third equation directly for A to get that
. We can then plug this into the first two equations to get,

72
So, the partial fraction decomposition for this expression is,

(b)
Here is the form of the partial fraction decomposition for this part.

Adding the two terms up gives,

Now, set the numerators equal and we might as well go ahead and multiply the right side out and
collect up like terms while we’re at it.

73
Setting coefficients equal gives,

In this case we got A and B for free and don’t get excited about the fact that
. This is not a problem and in fact when this happens the remaining work is often a little
easier. So, plugging the known values of A and B into the remaining two equations gives,

The partial fraction decomposition is then,

Partial fractions are very important and you really need to make sure that you understand the whole
procedure of breaking down the fraction into smaller fractions. It’s always examinable and very useful
in calculus. Notice that we have given you a good number of examples with detailed solutions to make
sure you understand.

74
INEQUALITIES
Expressions involving inequality signs such as <, >, or ,  are called inequalities. These type
of inequalities do not demand one value as an answer but a set of values which will satisfy the
inequality.

3.8.1 Linear Inequalities

Inequalities of the form ( x  a )  0 or ( x  a )  0 are called linear inequalities. Our problem is


to find the set of values that satisfied the inequality.

Example 3.8.1 Find the solution set for the following inequality, ( x  1)  0 .

Solution: We solve the inequality as follows:


( x  1)  0
x  1

The solution set is x : x  1, x  R, this is written in set builder notation

3.8.2 Quadratic Inequalities.

Inequalities of the form ax 2  bx  c  0 are called quadratic inequalities because they involve
quadratics. Inequalities of this form require you to factorize the expression on the left hand side.
After factorizing then you follows the following procedures in order for you get the set(s) of
values that satisfies the inequality.

Example 3.8.2 Find the solution set for the inequality x 2  3 x  2  0

Solution: You begin by factorizing the quadratic expression as follows;


x 2  3x  2  0
( x  2)( x  1)  0
To find the solution set of this inequality, you find the critical values of the inequality by solving
for x. In this case x = -2 or -1. You make a table showing the critical points as shown below:
Take a number in the given interval and substitute in the factor and write the sign in the box as
indicated, and the last row which is the product of the two factors will determine the solution
depending on the inequality given in the question. In this case, you want less than or equal so
your solution will be the interval with a negative in the last row.

75
The table above shows that the sets of point which satisfies the given quadratic inequality is read
from the last row, ie the shaded part, which is x : 2  x  1, x  R .

This does not matter how many factors that are being multiplied, all what is needed is to
find the critical value of each factor and this will give regions where to check whether the
factor is positive or negative and assign them to each region and follow the procedure as
given above.

Example. Find the solution set of ( x  1)( x  1)( x  2)  0


Solution: Here we find the values of x for which this inequality vanishes. These values are called
critical values. Here we have x = 1, -1, and 2.

So we make a table as follows:

The solution set to the given inequality is { x : 1  x  1, x  R}  { x : x  2, x  R}

76
x 1
Example 3.8.3 Find the solution set for the following inequality 2
x 1

Solution. To find the solution set of this problem, you first transpose 2 to the left because you
cannot cross multiply since the denominator is a variable. Note that when you multiply by a
negative quantity the sign changes, so x is a variable and it can assume a negative number or a
positive number, but we do not know for which values this x stands for as it is a variable. So we
proceed as follows:
x 1
2
x 1
x 1
20
x 1
x  1  2( x  1)
0
x 1
x  1  2x  2
0
x 1
 x3
0
x 1
x3
0
x 1

Note the change of the sign from greater than zero to less than zero.

You now find the critical values of the numerator and denominator and proceed as example 1
given above. The critical values are: x  3 and x  1

We hope you have enjoyed inequalities, they are the simplest part of this mathematics but yet
very useful at all times in life and so they are always examinable.

77
Binomial Theorem
Introduction

This section of work is to do with the expansion of expressions of the forms: (a  b) n where
a , b  R , n  Z  . Pascal's Triangle and the Binomial Theorem give you a way of expressing the
expansion as a sum of ordered terms. You will also discuss expressions of the form: (1  x) n
where x is a variable and n is a rational number or negative integers.

Pascal's Triangle

You know the expansion of the following:

(a  b) 0  1
( a  b )1  ab
(a  b) 2  a 2  2 ab  b 2
(a  b) 2  a 3  3a 2b  3ab 2  b 3
etc

See the coefficients of the terms given in each of the expressions given above. The coefficients
follow the following patterns:

The first line represents the coefficients for n=0. (a+b)0= 1 . The second line represents the
coefficients for n=1. (a+b)1= a + b. The third line represents the coefficients for n=2.
(a  b) 2  a 2  2ab  b 2 . The sixth line represents the coefficients for n=5.
(a  b)5  a 5  5a 4b  10 a 3b 2  10 a 2b3  5ab 4  b5 .

This is a method of predicting the coefficients of the binomial series is called Pascal triangle,
since the coefficients form a triangle. Coefficients are the constants (1,2,3,4,5,6 etc.) that
multiply each variable, or group of variables. Consider (a+b)n variables a, b, and n a positive
integer. The Binomial Theorem builds on Pascal's Triangle in practical terms, since writing out
78
triangles of numbers has its limitations. It is generally difficult to use Pascal’s triangle for n
greater than 6, hence the use of Binomial Theorem becomes necessary. For example (a  b) 20 ,
this would require you to tabulate the coefficients from 2 to 19 in order to obtain the coefficients
of (a  b) 20 . It is now necessary to state the Binomial Theorem which will help you to expand
expressions like (a  b) 20 . In using this theorem, you need to know combinations and to
appreciate this; a theorem may make it easy to follow:

n
Theorem: The total number of combinations of n objects taken r at a time C rn    is defined
r
n n!
as,    , this formula is valid for positive values n  Z , and n  r . This formula gives
 r  ( n  r )! r!
the coefficients of the Binomial expansion. The following notations are used for the formula
given above. n! n ( n  1)( n  2)( n  3)....... 1 , that is, 3! 3.2.1  6 , 2! 2.1  2 , 1! 1 , and 0! 1
5 5! 5! 5.4.3!
by definition. Therefore,       10 .
 3  (5  3)!3! 2!3! 2.1.3!

Theorem: (The Binomial Theorem): For any positive integer n,


n n n n n
( a  b ) n    a n    a n 1b    a n 2 b 2  .....    a n r b r  ....   b n where 'n' is the
0 1 2 r n
power/index of the original expression and 'r' is the number order. You can immediately note the
following properties of the expansion given above for any positive integer n.

(1) The number of terms in any identity is n+1.


n n
(2) The first term in the identity is a and the last term is b
(3) The exponent of a decreases by one and that of b increases by one from any term to the next
term, so that the sum of the exponents of a and b in any term is n.
(4) For the first term and last, the second and next to the last, the third and from the last, and so
forth, terms of the identity, the coefficients are the same.

This is a way of finding all the terms of the expansion, the coefficients and the powers of the
variables.

Our interest is generally to find the nth term; all you need is to understand the equation given
above. Note that term 1 has the power of b zero and r = 0, second term has power of b as 1 and r

79
= 1, third term has power of b as 2 and r = 2 etc. You can determine the nth term of the expansion
using this theory.

Example 3.9.1 Expand the expression (a  b)5 . You proceed as follows:

Example 3.9.2 Write down (3 x  2)5 as a Binomial expansion, using


(a  b)5  a 5  C (5,1)a 4b  C (5,2)a 3b 2  C (5,3)a 2b3  C (5,4)ab 4  b5

And the value of the coefficients from the previous, let a = 3x, b = -2
(3 x  2)  (3 x )  C (5,1)( 3 x ) ( 2)  C (5,2)( 3 x ) ( 2)  C (5,3)( 3 x ) ( 2)  C (5,4)( 3 x )( 2)  ( 2) 5
5 5 4 3 2 2 3 4

(3 x  2) 5  243 x 5  5(81 x 4 )( 2)  10 ( 27 x 3 )( 4)  10 (9 x 2 )( 8)  5(3 x )(16 )  ( 32 )


(3 x  2) 5  243 x 5  810 x 4  1080 x 3  720 x 2  240 x  32
Find the 15th term in the expansion of 3  2 x 
20
Example: 3.9.3

n
Solution: Using the general term   a n  r b r , you know n  20 , if you want the 15th term, this
r
means that r  14 , a  3 and b  2 x , substitute these values in the general form as:

 20  20 14
 3 ( 2 x )14
 14 
20! 6 14 14
3 .2 x
6!14!
20 .19 .18 .17 .16 .15 .14! 6 14 14
.3 .2 . x
6.5.4.3.2.1.14!
19 .17 .8.15 .36.214.x14

This is the 15th term of the given expansion.

80
16
 1
Example 3.9.4. Find the term independent of x in the expansion of  x  
 x

n
Solution: You write the general term   a n  r b r , in this case you know the value of n as n = 16,
r
1
you know the value of a as a = x and the value of b as b  , but you don’t know r, so you need
x
to find r. You substitute these values in the general form and solve for r, then substitute the value
of r and you will find the term which will have no x.

16  16  r 1 r
  x (  )
r x
16  16  r
  x ( 1) r x  r
r
16 
 ( 1) r x16  2 r
r

To get the term which is independent of x, mean set 16  2r  0 , because x0 is 1. Solving this
equation gives r  8 . That means the 9th term in this expansion is independent of x. When
substituting r = 8 in the general term and simplified the 9th term is obtained which is independent
of x.

16 
 ( 1)8 x16 16
8
16!
( 1)8 .1
8!8!
16 .15 .14 .13 .12 .11 .10 .9.8!
.1
8.7.6.5.4.3.2.1.8!
13 .11 .10 .9
12870

This is the 9th term which is independent of x in the expansion.

3.9.2 The Expansion of (1  x) n

The above method of expanding expressions of the form (a  b) n is only used when n is a
positive integer. For powers of rational numbers and negative integer, this method will not work.
Instead the following formula is used:

81
nx n(n  1) x 2 n(n  1)( n  2) x 3
(1  x) n  1 
   ....... For n  Z  and Q This
1! 2! 3!
produces an infinite series and this series is valid in the region x  1 . This series should be
written generally up to including the term containing x3, and write the region where this series is
valid.

Example Expand the following up to including the term x3

(i) (1  3 x ) 3 (ii) (1  2 x) 1

Example 3.9.5 Write the expansion of the expression 2  3 x , and write the region where this
expansion is valid.

Solution: This expression can be written as: ( 2  3 x ) 2 , so we can compare this expression with
the form given above. That means we write it in the form; (1  x) n and compare, and what you
1 1
3 3
see is that ( 2  3 x ) 2 can be written as; [ 2(1  (  x )] 2 Now we can compare as: x   x , and
2 2
1 nx n(n  1) x 2
n(n  1)( n  2) x 3
n substitute these in the expression (1  x)  1 
n
   .......
2 1! 2! 3!
3 1
and this is what we get; so using x x and n we have
2 2
1 1 1 1 1
1 (  1) (  1)(  2)
3 1
[ 2(1  (  x )]  2[1  x  2 2
2
x2  2 2 2 x 3  .........]
2 2 2! 3!

1
3 1 1 1 3
[ 2(1  (  x )] 2  2 [1  x  x 2  x 3  .........] This series is valid in the region x  1 which
2 2 8 16 2
2
implies x 
3

The pascal’s triangle as fun as it may look, is very important. It holds the whole Binomial
topic because it gives us the coefficients of the expansions, it lessens our work so that we may
be able to do things in a simpler way. Make sure you have understood how to get the nth term,
the term independent of x or any variable that you are asked. You can try the following exam
samples.

Exercise 3
1. Solve the following quadratic equations by factorization method.
(i) x2 – 4x – 12 = 0 (ii) 3x2 + 5x – 12 = 0

82
(iii) 3(4x – 1)2 + 1 = 16 (iv) (x – 2 )2 = – 4
2. Use the method of completing the squares to solve each of the following equations.
(i) x2 – 10x + 24 = 0 (ii) 2x2 + 12x + 5 = 0
(iii) 3x2 + 12 – 2 = 0 (iv) x2 – 3x = – 1
3. Without solving the equation, determine the nature of roots of the following equations.
(i) 4x2 + 20x + 25 = 0 (ii) x2 + 4x + 7 = 0
(iii) 16x2 = 40x – 25 (iv) 2x2 + 5 + 7 = 0

4. If  and  are the roots of the equation x2 – 4 x + 2 = 0 , find the value of :

1 1
(i)  (ii) (   1) (  1) (iii)  2  2
 

1 1
(iv)  2  2 (v) (    )2 (vi) 2
 2
 1  1

5. If  and  are the roots of the equation x2 – 4x + 2 = 0 , find equations whose roots
are; (i)  + 3 ,  + 3 (ii)  + 3 ,  + 3
6. (a) Find two numbers whose sum is 16 and whose product is a maximum.
(b) A man with 180m of fencing wishes to fence off an area in the shape of a
rectangle. What should be the dimension of the area if the enclosed space is to be as
large as possible?
7. For each of the following functions, find the maximum or minimum point and value.
Sketch the graph
(i) f(x) = x2 +6x + 5 (ii) f(x) = x2 + x – 6
(iii) f(x) = 2x2 + 5x – 12 (iv) f(x) = – 2x2 + 11x – 15
(iv) f(x) = – 3x2 + 5x – 4
8. Suppose that the equation p( x)  2 x 2  280 x  1000 , where x represents the number of
items sold, describes the profit function for a certain business. How many items should be
sold to maximize the profit?
9. The height of a projectile fired vertically into the air (neglecting air resistance) at an initial
velocity of 96 feet per second is a function of time x and is given by the equation
f ( x)  96 x  16 x 2 . Find the highest point reached by the projectile.
10. Find the value of k so that the equation:
(i) 2 x 2  kx  15  0 has one root = 3
(ii) 3 x 2  kx  2  0 has roots whose sum is equal to 6

83
1
(iii) 5 x 2  8 x  k  0 has roots whose product is equal to
5
(iv) 3 x 2  5 x  8  kx has roots numerically equal but opposite in sign.
11. (a) Find the range of values of k so that the equation:
(i) 3 x 2  4 kx  k  0 will have real roots
(ii) kx 2  4 3 x  k  1 will have imaginary roots
(b) Find the range of values of k or value of k so that the graph of the function where y
equals:
(i) 3 x 2  9 x  k will touch (have its vertex on ) the x axis.
3
(ii) x 2  2 kx   k will not intersect the x axis`
4
(iii) 4 x 2  4 2kx  k  3 will intersect the x axis inn two real points.
12. Find the remainder when
(a) 5 + 6x + 7x2 – x3 is divided by x + 2
(b) 6x3 + 7x2 – 15x + 4 is divided by x – 1
(c) x4 – 3x3 + 2x2 + 5 is divided by x – 1
(d) 8x3 – 10x2 + 7x + 3 is divided by 2x – 1
(e) 9x3 + 4 is divided by 3x + 2
13. Factorize the following polynomials
(a) 6x3 – 5x2 – 17x + 6
(b) 2x4 + 7x3 – 17x2 – 7x + 15
(c) 6x4 + 31x3 + 57x2 + 44x + 12
14. By using Synthetic Division, find the quotient and remainder when
(a) x4 – 2x3 – 3x2 – 4x – 8 is divided by (i) x – 2 (ii) x + 1
(b) x3 + 3x2 – 2x – 5 is divided by (i) x + 2 (ii) x - 3
15. Show that x + y is a factor of x5 + y5 and x7 + y7. By using synthetic division, find the
quotient in each case.
16. Factorize the polynomial x3 – 3x2 + 4x + 12. Hence calculate the ranges of values of x for
which x3 – 3x2  - 4x – 12.
17. Factorize the expression 6x3 – 7x2 – x + 12. Hence calculate the ranges of values of x for
which 6x3 – 7x2 > 12 – x .
18. Given that ( x + 2 ) is a factor of 2x3 + 6x2 + 6x – 5, find the remainder when the
expression is divided by ( 2x – 1 ) .
19. The expression 3x3 + 2x2 – bx + a is divisible by ( x – 1 ) but leaves a remainder of 10
when divided by ( x + 1 ). Find the values of a and b.

84
20. The expression 6x3 – 23x2 + ax + b gives a remainder of 11 when divided by ( x – 3 )
and a remainder of – 21 when divided by ( x + 1 ). Find the values of a and b and
hence factorize the expression.
1
2
21. Find the first four terms in the series expansion of (1  ) 2 in ascending power of x.
x

22. The series expansion of (1  px) q in ascending powers of x has coefficients -10 and 75 in x
and x 2 terms respectively.
(a) Find the values of p and q
(b) Find the coefficient of the x 3 and x 4 terms in the expansion
(c) State the set of values of x for which the series is valid.
23. Expand the following up and including the term x 3
3
1
(a) (1  2 x ) 2 (b) 4  3x (c)
1 x
1 20
24. Find the 15th term in the expansion ( x  ) . Hence find the tem independent of x.
2x

25. Find the values of x for which the following inequalities are satisfied:

(i) 3x-7 > 0 (ii) 5x-3 < 8x-12 (iii) x 2  2 x  99


x2 1 1
(iv) x 2  2x  4  0 (v) 0 (vi) 
x5 x2 3

TRIGONOMETRIC FUNCTIONS
4.1.1 Introduction

The word trigonometric was derived from two Greek words meaning measurement of triangles.
Historically the development of trigonometry began with the study of various relationships that
exists between the angles and the sides of triangles. This aspect of trigonometry has many
applications in surveying, navigation, carpentry etc.

4.1.2 Position angles

Consider a unit circle given below

85
(a) In the first quadrant : 0  x  90 
NP 
(i) sin       sine is positive in the first quadrant
OP 
NP 
(ii) cos       cosine is positive in the first quadrant
OP 
NP 
(iii) tan       tangent is positive in the first quadrant
ON 
(b) In the second quadrant: 90   x  180 
NP 
(i) sin       sine is positive in the first quadrant
OP 
NP 
(ii) cos       cosine is negative in the first quadrant
OP 
NP 
(iii) tan       tangent is negative in the first quadrant
ON 
(c) In the third quadrant: 180   x  270 
NP 
(i) sin       sine is negative in the first quadrant
OP 
NP 
(ii) cos       cosine is negative in the first quadrant
OP 
NP 
(iii) tan       tangent is positive in the first quadrant
ON 
(d) In the fourth quadrant: 270   x  360 
NP 
(i) sin       sine is negative in the first quadrant
OP 

86
NP 
(ii) cos       cosine is positive in the first quadrant
OP 
NP 
(iii) tan       tangent is negative in the first quadrant
ON 
TIP: Make sure you understand and master the sign of any trigonometric ratio in
each quadrant, don’t move till you have done so
Other Trigonometric Ratios

The other common trigonometric ratios are:

Other ratios are defined by using the above three:

cosecant

secant

cotangent

These six ratios define what are known as the trigonometric (trig in short) functions. They are
independent of the unit used.

4.1.3 Acute angle

Definition: Let  be any angle in standard position with its terminal sides in one of the four
quadrants. The reference angle associated with  is the acute angle formed by the terminal sides
of  and the x-axis.

The acute angle is determined by the terminal line with respect to the x-axis.

87
This terminal line can be in any quadrant, and take note on whether the trigonometric function in
the quadrant were the terminal line is positive or negative. The angles measured in anticlockwise
direction are positive while those measured in a clock wise direction are negative.

When locating a terminal line using angle the domain is normally stated to avoid arriving at the
same point infinite number of times. The terminal line 60  can be achieved 360 n  60  ,where
n  1,2,3,.......... . To avoid this, trigonometric functions specifies the domain as:
[180   x  180  ] or [0  x  360  ] .

4.1.4 The Pythagoras Theorem and other right angled triangle relationship

There are right angled relationships that are used extensively in trigonometry. One of the most
important of these relationship is the Pythagoras Theorem. If a right angled triangle has a and b
as measurements of the two sides and c is the measure of the hypotenuse. Then the relationship
between the sides are: c 2  a 2  b 2

Example Find the length of each side of an isosceles right angled triangle that has a
hypotenuse measurement as 6cm.

Solution Sketch the triangle

88
Applying the Pythagoras theorem

x2  x2  62
2 x 2  36
x 2  18
x  3 2

Therefore, each side is 3 2 cm long. Ignore the negative value.

These ratios are independent of the unit used to measure the sides as long as the same unit is
used for all the sides.

In particular, if we take H = 1, then

O = sin θ and A = cos θ.

Special angles

The trigonometric ratios of the angles 30º, 45º and 60º are often used in mechanics and other
branches of mathematics. So it is useful to calculate them and know their values by heart.. These

89
are called special angles found in the first quadrants and can be found in the second, third and
fourth quadrants as multiples of these angles.

60º & 30º

A very useful property comes directly from an equilateral triangle. Consider the equilateral
triangle with sides 2units. Draw an equilateral triangle, ABC, of sides 2 units in length. Next
draw a line AD from A perpendicular to BC. AD bisects BC giving BD = CD = 1.

From elementary geometry, an angle bisector of an equilateral is also a perpendicular bisector of


the opposite sides. Therefore, in figure 4.1.3 above the angle bisector divides the equilateral
triangle into two right angled triangles having acute angles of 30  , and 60  . The following
general property can be stated as: In a 30  , and 60  right angled triangle , the length of the side
opposite the 30  angle is equal is equal to one-half of the length of the hypotenuse.

The other triangle useful in trigonometry is an isosceles triangle with acute angles forty five
degrees.

45º

In this case, the triangle is isosceles. Hence the opposite side and adjacent sides are equal, say 1
unit.
The hypotenuse is therefore of length units (by Pythagoras Theorem).

90
These two right angled triangles help to find exact values of trigonometry functions.

These angles are called special angles because trigonometric functions of these angles can be
evaluated without a calculator or tables. These angles can either be expressed in degrees or
radians.

91
Example 4.1.1 Without using calculator, find the exact value of:

(i) sin 135 


Solution: Locate the terminal line for the angle 135  . Note that this angle is in the second
quadrant.

92
The terminal line is in the second quadrant and sine in this quadrant is positive, so the
1
answer is positive. So sin 135   sin 45   .
2

Note that the value of the trigonometric function can be evaluated by the use of acute
angles with the x-axis.

(ii) tan 210 


Solution: Locate the terminal line for the angle 210  . Note that this angle is in the
quadrant.

The terminal line is in the third quadrant and tan in this quadrant is positive, so the
1
answer is positive. So tan 210   tan 30   .
3

93
3
(iii) cos
4

(iv) tan
3
2
(v) cot(  )
3
11
(vi) sec(  )
6
(vii) sin( 60  )
(viii) cos(210  )
TIP: At least you should know all the special angles, the values of each trig ratio at each
special angle, it will help you understand the graphs easily.

Graphs of Trigonometric functions and reciprocal functions.

In a right triangle, there are actually six possible trigonometric ratios, or functions.
A Greek letter (such as theta or phi ) will now be used to represent the angle.

You've already learned the basic trig graphs. But just as you could make the basic quadratic x2,
more complicated, such as –(x + 5)2 – 3, so also trig graphs can be made more complicated. We
can transform and translate trig functions, just like you transformed and translated other
functions in algebra.

Let's start with the basic sine function, f(t) = sin(t). This function has an amplitude of 1 because
the graph goes one unit up and one unit down from the midline of the graph. This function has a
period of 2π because the sine wave repeats every 2π units. The graph looks like this:

Now let's look at g(t) = 3sin(t):

94
Do you see that the graph is three times as tall? The amplitude has changed from 1 to 3. This is
always true: Whatever number A is multiplied on the trig function gives you the amplitude; in
this case, that number was 3. So 0.5cos(t) would have an amplitude of 1/2, and –2cos(t) would
have an amplitude of 2 and would also be flipped upside down.

Now let's look at h(t) = sin(2t): Copyright © Elizabeth Stapel 2010 All Rights Reserved

Do you see that the graph is squished in from the sides? Do you see that the sine wave is cycling
twice as fast, so its period is only half as long? This is always true: Whatever value B is
multiplied on the variable, you use this value to find the period ω (omega) of the trig function,
according to this formula:

general period formula:

For sines and cosines (and their reciprocals), the "regular" period is 2π, so the formula is:

95
period formula for sines &
cosines:

For tangents and cotangents, the "regular" period is π, so the formula is:

period formula for tangents &


cotangents:

In the sine wave graphed above, the value of B was 2. (Sometimes the value of B inside the
function will be negative, which is why there are absolute-value bars on the denominator.) The
formula for sines and cosines says that cos(3t) will have a period of (2π)/3 = (2/3)π; on the other
hand, the formula for tangents and cotangents says that tan(t/2) will have a period of (2π)/(1/2) =
4π.

(Note: Different books use different letters to stand for the period formula. In your class, use
whatever your book or instructor uses.)

Now let's looks at j(t) = sin(t – π/3):

Do you see that the graph (in blue) is shifted over to the right by π/3 units from the regular graph
(in gray)? This is always true: If the argument of the function (the thing you're plugging in to the
function) is of the form (variable) – (number), then the graph is shifted to the right by that
(number) of units; if the argument is of the form (variable) + (number), then the graph is shifted
to the left by that (number) of units. So cos(t + π/4) would be shifted to the left by π/4 units, and
tan(t – 2π/3) would be shifted to the right by (2/3)π units. This right-or-left shifting is called
"phase shift".

Now let's look at k(t) = sin(t) + 3:

96
Do you see how the graph was shifted up by three units? This is always true: If a number D is
added outside the function, then the graph is shifted up by that number of units; if D is
subtracted, then the graph is shifted down by that number of units. So cos(t) – 2 is the regular
cosine wave, but shifted downward two units; and tan(t) + 0.6 is the regular tangent curve, but
shifted upward by 6/10 of a unit.

Putting it all together, we have the general sine function, F(t) = Asin(Bt – C) + D, where A is the
amplitude, B gives you the period, D gives you the vertical shift (up or down), and C is used to
find the phase shift. Why don't you always just use C? Because sometimes more is going on
inside the function. Remember that the phase shift comes from what is added or subtracted
directly to the variable. So if you have something like sin(2t – π), the phase shift is not π units!
Instead, you first have to isolate what's happening to the variable by factoring: sin(2(t – π/2)).
Now you can see that the phase shift will be π/2 units, not π units. So the phase shift, as a
formula, is found by dividing C by B.

For F(t) = Af(Bt – C) + D, where f(t) is one of the basic trig functions, we have:

o A: amplitude is A
o B: period is (2π)/|B|
o C: phase shift is C/B
o D: vertical shift is D

Let's see what this looks like, in practice, because there's a way to make these graphs a whole lot
easier than what they show in the book....

 Graph two periods of g(x) = sin(πx + π/2) + 3

The amplitide is going to be the same, but the midline of the graph is going to be at y = 3
instead of y = 0 (that is, the x-axis). The period is going to be (2π)/(π) = 2. The argument
factors as π(x + 1/2), so the graph will be shifted 1/2 unit to the left.

97
I'll do the usual graph: Copyright © Elizabeth Stapel 2010-2011 All Rights Reserved

Instead of shifting the sine wave three units up, I'll shift the horizontal axis three units
down, and re-number the y-axis:

The regular period goes from 0 to 2π; this one goes from 0 to 2, so I'll re-number the x-
axis:

98
And the graph is shifted to the left by 1/2, so I'll shift the y-axis to the right by 1/2 and
re-number the x-axis a bit more:

Can you see why I used pencil and did a lot of erasing when I was doing graphing?

My best advice regarding doing these graphs is to practice, practice, practice. You don't want to
freeze or have a "brain-fart" on the test, and this is pretty straightforward, once you get the feel
of it. So keep doing extra graphing, until you are feeling comfortable and confident in your
skills.

In mathematics, an "identity" is an equation which is always true. These can be "trivially" true,
like "x = x" or usefully true, such as the Pythagorean Theorem's "a2 + b2 = c2" for right triangles.
There are loads of trigonometric identities, but the following are the ones you're most likely to
see and use.

Basic & Pythagorean, Angle-Sum & -Difference, Double-Angle, Half-Angle, Sum, Product

Basic and Pythagorean Identities

99
Notice how a "co-(something)" trig ratio is always the reciprocal of some "non-co" ratio. You
can use this fact to help you keep straight that cosecant goes with sine and secant goes with
cosine.

sin2(t) + cos2(t) = 1 tan2(t) + 1 = sec2(t) 1 + cot2(t) = csc2(t)

The above, because they involve squaring and the number 1, are the "Pythagorean" identities.
You can see this clearly if you consider the unit circle, where sin(t) = y, cos(t) = x, and the
hypotenuse is 1.

sin(–t) = –sin(t) cos(–t) = cos(t) tan(–t) = –tan(t)

Notice in particular that sine and tangent are odd functions, while cosine is an even function.

Angle-Sum and -Difference Identities

sin(α + β) = sin(α)cos(β) + cos(α)sin(β)


sin(α – β) = sin(α)cos(β) – cos(α)sin(β)
cos(α + β) = cos(α)cos(β) – sin(α)sin(β)
cos(α – β) = cos(α)cos(β) + sin(α)sin(β)

Double-Angle Identities

sin(2x) = 2sin(x)cos(x)

cos(2x) = cos2(x) – sin2(x) = 1 – 2sin2(x) = 2cos2(x) – 1

Half-Angle Identities Copyright © Elizabeth Stapel 2010-2011 All Rights Reserved

100
The above identities can be re-stated as:

sin2(x) = ½[1 – cos(2x)]

cos2(x) = ½[1 + cos(2x)]

Sum Identities

Product Identities

You will be using all of these identities, or nearly so, for proving other trig identities and for
solving trig equations. However, if you're going on to study calculus, pay particular attention to
the restated sine and cosine half-angle identities, because you'll be using them a lot in integral
calculus.

Solving trig equations use both the reference angles you've memorized and a lot of the algebra
you've learned. Be prepared to need to think!

 Solve sin(x) + 2 = 3 for 0° < x < 360°

Just as with linear equations, I'll first isolate the variable-containing term:

101
sin(x) + 2 = 3
sin(x) = 1

Now I'll use the reference angles I've memorized:

x = 90°

 Solve tan2(x) + 3 = 0 for 0° < x < 360°

There's the temptation to quickly recall that the tangent of 60° involves the square root of
3 and slap down an answer, but this equation doesn't actually have a solution:

tan2(x) = –3

How can the square of a trig function evaluate to a negative number? It can't!

no solution

 Solve on 0° < x < 360°

To solve this, I need to do some simple factoring:

Now that I've done the algebra, I can do the trig. From the first factor, I get x = 90° and x
= 270°. From the second factor, I get x = 30° and x = 330°.

x = 30°, 90°, 270°, 330° Copyright © Elizabeth Stapel2010-2011 All Rights


Reserved

 Solve sin2(x) – sin(x) = 2 on 0° < x < 360°

This is a quadratic in sine, so I can apply some of the same methods:

sin2(x) – sin(x) – 2 = 0
(sin(x) – 2)(sin(x) + 1) = 0
sin(x) = 2 (not possible!) or sin(x) = –1

Only one of the factor solutions is sensible. For sin(x) = –1, I get:

102
x = 270°

 Solve cos2(x) + cos(x) = sin2(x) on 0° < x < 360°

I can use a trig identity to get a quadratic in cosine:

cos2(x) + cos(x) = sin2(x)


2
cos (x) + cos(x) = 1 – cos2(x)
2cos2(x) + cos(x) – 1 = 0
(2cos(x) – 1)(cos(x) + 1) = 0
cos(x) = 1/2 or cos(x) = –1

The first trig equation, cos(x) = 1/2, gives me x = 60° and x = 300°. The second equation
gives me x = 180°. So my complete solution is:

x = 60°, 180°, 300°

 Solve sin(x) = sin(2x) on 0° < x < 360°

I can use a double-angle identity on the right-hand side, and rearrange and simplify; then
I'll factor:

sin(x) = 2sin(x)cos(x)
sin(x) – 2sin(x)cos(x) = 0
sin(x)(1 – 2cos(x)) = 0
sin(x) = 0 or cos(x) = 1/2

I can The sine wave is zero at 0°, 180°, and 360°. The cosine is 1/2 at 60°, and thus also
at 360° – 60° = 300°. So the complete solution is:

x = 0°, 60°, 180°, 300°, 360°

 Solve sin(x) + cos(x) = 1 on 0° < x < 360°

Hmm... I'm really not seeing anything here. It sure would have been nice if one of these
trig expressions were squared...

Well, why don't I square both sides, then, and see what happens?

(sin(x))2 + cos(x))2 = (1)2


sin2(x) + 2sin(x)cos(x) + cos2(x) = 1
[sin2(x) + cos2(x)] + 2sin(x)cos(x) = 1
1 + 2sin(x)cos(x) = 1
2sin(x)cos(x) = 0
sin(x)cos(x) = 0

103
Huh; go figger: I squared, and got something that I could work with. Nice!

From the last line above, either sine is zero or else cosine is zero, so my solution appears
to be:

x = 0°, 90°, 180°, 270°

However (and this is important!), I squared to get this solution, so I need to check my
answers in the original equation, to make sure that I didn't accidentally create solutions
that don't actually count. Plugging back in, I see:

sin(0°) + cos(0°) = 0 + 1 = 1 (this solution works)


sin(90°) + cos(90°) = 1 + 0 = 1 (this one works, too)
sin(180°) + cos(180°) = 0 + (–1) = –1 (oh;okay, so this one does NOT work)
sin(270°) + cos(270°) = (–1) + 0 = –1 (this one doesn't work, either)

So the actual solution is:

x = 0°, 90°

Note that I could have used the double-angle identity for sine, in reverse, instead of dividing off
the 2 in the next-to-last line in my computations. The answer would have been the same, but I
would have needed to account for the solution interval:

2sin(x)cos(x) = sin(2x) = 0

Then 2x = 0°, 180°, 360°, 540°, etc, and dividing off the 2 from the x would give me x = 0°, 90°,
180°, 270°, which is the same almost-solution as before. After doing the necessary check
(because of the squaring) and discarding the extraneous solutions, my final answer would have
been the same as before.

This squaring trick doesn't come up often, but if nothing else is working, it might be worth a try.

 Solve sin(x/2) = cos(x/2) in full generality.

There are various ways of going about this, but I think I'll take an easy way out: By
dividing through by the cosine, I'll get a tangent:

tan(x/2) = 1

(This division was okay, because I know from the graphs that the cosine isn't equal to
zero anywhere that it's equal to the sine of the same angle.)

The tangent is equal to 1 for x/2 = 45° and 225° on the first period. But this exercise
wants the answer "in full generality". Obviously, I can't list out all of the solution values,
because there are infinitely many. So I'll have to use a formula.

104
From what I know about the graph of the tangent, I know that the tangent will equal 1 at
45° after every 180°. These solutions for tan(x/2) are at 0° + 45°, 180° + 45°, 360° + 45°,
and so forth. To give the answer "in full generality", I'll use a formula:

x/2 = (180×n)° + 45°, for all integers n

Now I need to solve for x itself. I'll multiply through by 2:

x = (360n)° + 90°

 Solve 3tan3(x) – 3tan2(x) – tan(x) + 1 = 0 in full generality.

I can factor this in pairs:

3tan2(x)[tan(x) – 1] – 1[tan(x) – 1] = 0
[tan(x) – 1][3tan2(x) – 1] = 0
2
tan(x) = 1 or tan (x) = 1/3
tan(x) = 1 or tan(x) = ±1 / sqrt[3]

The first equation solves as x = 45° and 225°; the second solves as x = 30°, 150°, 210°,
and 330°. To make the solution "general", I need to state these formulaically.

The first solution is 45° more than a multiple of 180°, so (180n)° + 45° should do. The
second solution is 30° more than a multiple of 180° and (because of the "plus / minus")
also 30° less than the same multiple, so (180n)° ± 30° will cover this part.

x = (180n)° ± 30°, (180n)° + 45° for all integers n

 Solve on

What on earth...?

When nothing works, sometimes it helps to put everything in terms of sine and cosine.
That process, applied to this equation, gives me:

That's not a whole lot better... but the first two terms share a common factor of 2. If I
convert the last term to a common denominator with the third term, what will that give
me?

105
If I factor a 2 from the first two terms and the square root of 3 and a cosine from the
second two terms, I'll get:

Now I can take the common factor out front:

Whew! That actually worked! Okay, now I need to solve the factors. The first factor
solves as:

Copyright © Elizabeth Stapel 2010-2011 All Rights


Reserved

This equation is true at x = 60° and, by the symmetry of the tangent curve, also at x =

180° + 60° = 240°. In radians, this is . And the second factor solves as:

Cosine takes on this value at x = 30° and, by the symmetry of the cosine curve, also at x =

360° – 30° = 330°. In radians, this is . So my solution is:

 Solve ln(2 – sin2(x)) = 0 on 0° < x < 360°

The natural log is zero when the argument is 1, so this gives me:

2 – sin2(x) = 1
1 – sin2(x) = 0
(1 – sin(x)) (1+sin(x)) = 0

106
1 = sin(x)
1 = –sin(x)

From what I know of the sine wave, my solution is:

x = 90°, 270°

 Solve log3(2sin(x)) = 1/2 on [0, 2π)

By nature of logarithms, the equivalent exponential equation is:

The sine takes on this value at and also at . Then my solution is:

Expect to need to factor (especially quadratics) and to use trig identities. Don't be afraid to try
different methods; sometimes your first impulse doesn't lead anywhere helpful, but your second
guess might work fine. And pay particular attention to any oddly complex examples in your
textbook, as these may hold hints about what tricks you will need, especially on the test.

Properties of Trigonometric Functions

The properties of the 6 trigonometric functions: sin (x), cos (x), tan(x), cot (x), sec (x) and csc (x) are
discussed. These include the graph, domain, range, asymptotes (if any), symmetry, x and y intercepts
and maximum and minimum points.

Sine Function : f(x) = sin (x)

 Graph

107
 Domain: all real numbers

 Range: [-1 , 1]

 Period = 2pi

 x intercepts: x = k pi , where k is an integer.

 y intercepts: y = 0

 maximum points: (pi/2 + 2 k pi , 1) , where k is an integer.

 minimum points: (3pi/2 + 2 k pi , -1) , where k is an integer.

 symmetry: since sin(-x) = - sin (x) then sin (x) is an odd function and its
graph is symmetric with respect to the origin (0 , 0).

 intervals of increase/decrease: over one period and from 0 to 2pi, sin


(x) is increasing on the intervals (0 , pi/2) and (3pi/2 , 2pi), and
decreasing on the interval (pi/2 , 3pi/2).

Cosine Function : f(x) = cos (x)

 Graph

108
 Domain: all real numbers

 Range: [-1 , 1]

 Period = 2pi

 x intercepts: x = pi/2 + k pi , where k is an integer.

 y intercepts: y = 1

 maximum points: (2 k pi , 1) , where k is an integer.

 minimum points: (pi + 2 k pi , -1) , where k is an integer.

 symmetry: since cos(-x) = cos (x) then cos (x) is an even function and its
graph is symmetric with respect to the y axis.

 intervals of increase/decrease: over one period and from 0 to 2pi, cos


(x) is decreasing on (0 , pi) increasing on (pi , 2pi).

Tangent Function : f(x) = tan (x)

 Graph

109
 Domain: all real numbers except pi/2 + k pi, k is an integer.

 Range: all real numbers

 Period = pi

 x intercepts: x = k pi , where k is an integer.

 y intercepts: y = 0

 symmetry: since tan(-x) = - tan(x) then tan (x) is an odd function and its
graph is symmetric with respect the origin.

 intervals of increase/decrease: over one period and from -pi/2 to pi/2,


tan (x) is increasing.

 Vertical asymptotes: x = pi/2 + k pi, where k is an integer.

Cotangent Function : f(x) = cot (x)

 Graph

110
 Domain: all real numbers except k pi, k is an integer.

 Range: all real numbers

 Period = pi

 x intercepts: x = pi /2 + k pi , where k is an integer.

 symmetry: since cot(-x) = - cot(x) then cot (x) is an odd function and its
graph is symmetric with respect the origin.

 intervals of increase/decrease: over one period and from 0 to pi, cot


(x) is decreasing.

 Vertical asymptotes: x = k pi, where k is an integer.

Secant Function : f(x) = sec (x)

 Graph

111
 Domain: all real numbers except pi/2 + k pi, n is an integer.

 Range: (-infinity , -1] U [1 , +infinity)

 Period = 2 pi

 y intercepts: y = 1

 symmetry: since sec(-x) = sec (x) then sec (x) is an even function and its
graph is symmetric with respect to the y axis.

 intervals of increase/decrease: over one period and from 0 to 2 pi, sec


(x) is increasing on (0 , pi/2) U (pi/2 , pi) and decreasing on (pi , 3pi/2) U
(3pi/2 , 2pi).

 Vertical asymptotes: x = pi/2 + k pi, where k is an integer.

Cosecant Function : f(x) = csc (x)

 Graph

112
 Domain: all real numbers except k pi, k is an integer.

 Range: (-infinity , -1] U [1 , +infinity)

 Period = 2pi

 symmetry: since csc(-x) = - csc(x) then csc (x) is an odd function and its
graph is symmetric with respect the origin.

 intervals of increase/decrease: over one period and from 0 to 2pi, csc


(x) is decreasing on (0 , pi/2) U (3pi/2 , 2pi) and increasing on (pi/2 , pi)
U (pi / , 3pi/2).

 Vertical asymptotes: x = k pi, where k is an integer.

TIP: All you need to know here is the behaviour of each trig function, where it is
negative, zero and positive. And also what happens as each approaches zero. Of course
you must know the domains, ranges, phase shifts and the amplitude. Then you are set
to go because in any exam, you must perform this.

113
Logarithm and Exponential functions
Introduction
This unit introduces special functions which plays major roles in calculus. It shows the
connectedness of these two functions which would help you when navigating from one base to
the other. Graphs of these functions are explored and show how these functions can help certain
types of equations.

4.3 Exponential functions

A function of the form f ( x)  e x where x is real and a is a positive constant is called an


exponential function. The word “exponential” comes from the word exponent, which is another
word for “power’ or “index’. So for an exponential function f ( x)  e x , x  R , the variable x is
call the power , or the index, or the exponent. For a = 2, 3, etc, you can see how the graph of
each of these functions look like.

The Natural Logarithm function

The graph of the function f ( x)  e x , x  R is as follows:

114
You can see that this function is one-to-one. So it must have an inverse. To find the inverse
function, write x  e y , and try to make y the subject of the equation. In the expression e y , e is
called the base and y is called the power, or index or exponent. A fourth name for y is the
logarithm. So the expression x  e y can be interpreted as meaning that y is the logarithm of the
number x in the base e. We write this as y = log 𝑒 𝑥 or more usually as y  ln x . Logarithm in
base e is usually called natural logarithm and it is written as: y  ln x sine natural logarithm of
0 and negative numbers is undefined. But without loss of understanding we generally write
y  ln x . The function g ( x )  ln x , x  0 , is the inverse of the function f ( x)  e x , x  R , that is
g  f 1 , and the graphs of f and its inverse are shown below:

These are the graphs of e x and of ln x . You can see that the graph of y  ln x is the reflection of
the graph of y  e x in the line y  x .

4.4 Logarithms

You have seen in the section above that the word power; index, exponent, and logarithm are
synonymous: they are four different words to describe exactly the same thing.

115
In this book we use the word “logarithm” and take the base to the positive ( a  0 ) , and the graph
will look like:

Now the statement b  a c reads ‘the number b is equal to the base a raised to the logarithm c’.
Another way of reading the statement is ‘c is the logarithm of the number b to the base a’. What
this means that you can express an exponential form into logarithm and vise-visa. That is,
a c  b  𝑐 = log 𝑎 𝑏 . This is a very useful relationship between exponential and logarithm when
changing bases of logarithms.

Laws of logarithm

1. log 𝑎 𝑀𝑁 = log 𝑎 𝑀 + log 𝑎 𝑁


𝑀
2. log 𝑎 𝑁 = log 𝑎 𝑀 − log 𝑎 𝑁
n
3. log a  n log a
M M

4. log aa  1
5. log 𝑎 1 = 0

TIP: Make sure you master the laws of logarithm and exponential. They are the key to
understanding this whole topic

2  x
Example 4.4.1 Find x if log 32

Solution: You write this in exponential form: log 2 32 = 𝑥  2 x  32 this the same as
 2 x  2 5 , by inspection you have x = 5

Example 4.4.2 Find the value of log 3243

Solution: This can be written as: log 3 243 = log 3 35 = 5log 3 3 = 5

Example 4.4.3 Simplify log 𝑎 4 + 2log 𝑎 3 + log 𝑎 32 − log 𝑎 6

Solution:: This can be written as:


4𝑥9𝑥9 324
log 𝑎 4 + log 𝑎 32 + log 𝑎 9 − log 𝑎 6=log 𝑎 = log 𝑎 = log 𝑎 54
6 6

Equations of the form a x  b

116
Although you can solve equation such as 3 x  9 , 4 x  64 , and so on, because the value of x is an
integer, in general you cannot solve such equations by inspection. It is time consuming to find a
solution to 5 x  67 by trial and improvement. The standard method of solving such equations is
by taking logarithms with suitable base.

Example 4.4.4 Solve the equation 2(52 x )  5 x  6

Solution: In solving such equations, you let z  5 x and your equation becomes 2 z 2  z  6

3
Which is a quadratic equation and when you solve this equation your solutions are z  2,  .
2
You ignore the negative since there is no logarithm of a negative number. So you use z = 2, from
this value you can find the value of x as follows: z  5 x implies 2  5 x taking logarithm to base
5 will give you 𝑥 = log 5 2.

Exercise 4
1. Solve the equations
(a) 5 3 X  2  43 (b) 4 3 x  2  7 x  3 (c) 25 x  5 x 1  6 (d ) 2e x  2e  x  5

(e) 2(3 2 x )  9(3 x )  4  0 (f) 4 2 x  48  4 x  2 (g) 2 2 x 1  3(2 x )  1

2. Find the values of x for which log 3x  2 log 3x  1

3. Solve the equations

(a) log 3 (2 − 3𝑥) = log 9 (6𝑥 2 − 19𝑥 + 2) (b) log 2x  4 log 2x  5

4. (i) Find the values of x for which 2 ln 2 x  6 ln 2  ln( x  3)

ln 16
(ii) Given that x + y = 2 and 3 x  4 y Show that x  , and find y in a similar
ln 12
form.

END OF VOLUME ONE.. HOPE YOU ENJOYED


IT. VOLUME TWO STARTS WITH CALCULAS.

117

You might also like